SlideShare a Scribd company logo
1. INTRODUCTION TO STATISTICS 
The impact of statistics profoundly affects society today. Every day, we have
confronted with some form of statistical information through news papers, magazines,
and other forms of communication. Besides, statistical tables, survey results, and the
language of probability are used with increasing frequency by the media. Such
statistical information has become highly influential in our lives.
Statistics can be considered as numerical statements of facts which are highly
convenient and meaningful forms of communication. The subjects of statistics, as it
seems, is not a new discipline but it is as old as the human society itself. The sphere of
its utility, however, was very much restricted. The word statistics is derived from the
Latin word status which means a political state or government. It was originally
applied in connection with kings and monarchs collecting data on their citizenry
which pertained to state wealth, taxes collected population and so on. Thus, the scope
of statistics in the ancient times was primarily limited to the collection of
demographic and property and wealth data of a country by governments for framing
military and fiscal policies.
Now days, statistics is used almost in every field of study, such as natural science,
social science, engineering, medicine, agriculture, etc. The improvements in computer
technology make it easier than ever to use statistical methods and to manipulate
massive amounts of data.
Definition and Classification of Statistics
Statistics as a subject (field of study): Statistics is defined as the science of
collecting, organizing, presenting, analyzing and interpreting numerical data to make
decision on the bases of such analysis.(Singular sense)
Statistics as a numerical data: Statistics is defined as aggregates of numerical
expressed facts (figures) collected in a systematic manner for a predetermined
purpose. (Plural sense)
In this course, we shall be mainly concerned with statistics as a subject, that is, as a
field of study.
Classification of Statistics: Statistics is broadly divided into two categories based on
how the collected data are used.
i. Descriptive Statistics: is concerned with summary calculations, graphs, charts and
tables. It deals with describing data without attempting to infer anything that goes
beyond the given set of data. It consists of collection, organization, summarization
and presentation of data in the convenient and informative way.
Examples:
• The amount of medication in blood pressure pills.
• The starting salaries for Mathematics and Statistics students in different
organizations.
1 
 
ii. Inferential Statistics: Descriptive Statistics describe the data set that’s being
analyzed, but doesn’t allow us to draw any conclusions or make any inferences
about the data, other than visual “It looks like …..” type statements. It deals with
making inferences and/or conclusions about a population based on data obtained
from a limited sample of observations. It consists of performing hypothesis testing,
determining relationships among variables and making predictions. For example,
the average income of all families (the population) in Ethiopia can be estimated
from figures obtained from a few hundred (the sample) families.
Examples:
a) From past figures, it has been predicted that 90 0
0 of registered voters will vote in the
November election.
b) The average age of a student in Hawassa University is 19.1 years.
c) To determine the most effective dose of a new medication (on the basis of tests
performed with volunteer patients from selected hospitals)
d) To compare the effectiveness of two reducing diets (based on the weights losses of
persons who were taking the diets)
e) There is a relationship between smoking tobacco and an increased risk of developing
cancer.
Stages of Statistical Investigation
The area of statistics incorporates the following five stages. These are collection,
organization, presentation, analysis and interpretation of data.
Proper collection of data:
Data can be collected in a variety of ways; one of the most common methods is
through the use of survey. Survey can also be done in different methods, three of the
most common methods are: Telephone survey, Mailed questionnaire, Personal
interview.
Organization of data: If an investigator has collected data through a survey, it is
necessary to edit these data in order to correct any apparent inconsistencies,
ambiguities, and recording errors.
Presentation of data: the organized data can now be presented in the form of tables
or diagrams or graphs. This presentation in an orderly manner facilitates the
understanding as well as analysis of data.
Analysis of data: the basic purpose of data analysis is to make it useful for certain
conclusions. This analysis may simply be a critical observation of data to draw some
meaningful conclusions about it or it may involve highly complex and sophisticated
mathematical techniques.
Interpretation of data: Interpretation means drawing conclusions from the data
which form the basis of decision making. Correct interpretation requires a high degree
of skill and experience and is necessary in order to draw valid conclusions.
2 
 
Definition of some basic terms
Population: It is the totality of things under considerations. The population represents
the target of an investigation, and the objective of the investigation is to draw
conclusions about the population hence we sometimes call it target population.
Examples:
• All university students of Ethiopia
• Population of trees under specified climatic conditions
• Population of animals fed a certain type of diet
• Population of farms having a certain type of natural fertility
• Population of households, etc
The population could be finite or infinite (an imaginary collection of units).
There are two ways of investigation: Census and sample survey.
Census: a complete enumeration of the population. But in most real problems it
cannot be realized, hence we take sample.
Sample: A group of subjects selected from a population. A sample from a population
is the set of measurements that are actually collected in the course of an investigation.
For example, if we want to study the income pattern of lecturers at Hawassa
University and there are 3000 lecturers, then we may take a random sample of only
250 lecturers out of this entire population of 3000 for the purpose of study. Then this
number of 250 lecturers constitutes a sample.
In practice, we rarely conduct census, instead we conduct sample survey
Parameter: is a descriptive measure of a population, or summary value calculated
from a population. Examples: Average, Range, proportion, variance,
Statistic: is a descriptive measure of a sample, or summary value calculated from a
sample. From the previous example, the summary measure that describes a
characteristic such as average income of this sample is known as a statistic.
Sampling: The process or method of sample selection from the population.
Sample size: The number of elements or observation to be included in the sample.
Variable: It is an item of interest that can take on many different numerical values.
Applications, Uses and Limitation of Statistics
Applications of Statistics
Statistics can be applied in any field of study which seeks quantitative evidence. For
instance, engineering, economics, natural science, etc.
a) Engineering: Statistics have wide application in engineering.
• To compare the breaking strength of two types of materials
• To determine the probability of reliability of a product.
• To control the quality of products in a given production process.
• To compare the improvement of yield due to certain additives
(fertilizer, herbicides, (wee decides), e t c
b) Economics: Statistics are widely used in economics study and research.
• To measure and forecast Gross National Product (GNP)
3 
 
• Statistical analyses of population growth, unemployment figures, rural
or urban population shifts and so on influence much of the economic
policy making
• Financial statistics are necessary in the fields of money and banking
including consumer savings and credit availability.
c) Statistics and research: there is hardly any advanced research going on
without the use of statistics in one form or another. Statistics are used
extensively in medical, pharmaceutical and agricultural research.
Uses of Statistics
Today the field of statistics is recognized as a highly useful tool to making decision
process by managers of modern business, industry, frequently changing technology. It
has a lot of functions in everyday activities. The following are some uses of statistics:
• Statistics condenses and summarizes complex data: the original set of
data (raw data) is normally voluminous and disorganized unless it is
summarized and expressed in few numerical values.
• Statistics facilitates comparison of data: measures obtained from
different set of data can be compared to draw conclusion about those sets.
Statistical values such as averages, percentages, ratios, etc, are the tools
that can be used for the purpose of comparing sets of data.
• Statistics helps in predicting future trends: statistics is extremely useful
for analyzing the past and present data and predicting some future trends.
• Statistics influences the policies of government: statistical study results
in the areas of taxation, on unemployment rate, on the performance of
every sort of military equipment, etc, may convince a government to
review its policies and plans with the view to meet national needs and
aspirations.
• Statistical methods are very helpful in formulating and testing hypothesis
and to develop new theories.
Limitations of Statistics
The field of statistics, though widely used in all areas of human knowledge and
widely applied in a variety of disciplines such as business, economics and research,
has its own limitations. Some of these limitations are:
a) It does not deal with individual values: as discussed earlier, statistics deals
with aggregate of values. For example, the population size of a country for
some given year does not help us for comparative studies, unless it can be
compared with some other countries or with a set standard.
b) It does not deal with qualitative characteristics directly: statistics is not
applicable to qualitative characteristics such as beauty, honesty, poverty,
standard of living and so on since these cannot be expressed in quantitative
terms. These characteristics, however, can be statistically dealt with if some
quantitative values can be assigned to these with logical criterion. For
4 
 
c) Statistical conclusions are not universally true: since statistics is not an
exact science, as is the case with natural sciences, the statistical conclusions
are true only under certain assumptions. Also, the field deals extensively with
the laws of probability which at best are educated guesses. For example, if we
toss a coin 10 times where the chances of a head or a tail are 1:1, we cannot
say with certainty that there will be 5 heads and 5 tails. Thus the statistical
laws are only approximations.
d) It is sensitive for misuse: The famous statement that figures don’t lie but the
liars can figure, is a testimony to the misuse of statistics. Thus inaccurate or
incomplete figures can be manipulated to get desirable references. The
number of car accidents committed in a city in a particular year by women
drivers is 10 while that committed by men drivers is 40. Hence women
drivers are safe drivers.
TYPES OF VARIABLES AND MEASUREMENT SCALES
Types of Variables
A variable is a characteristic of an object that can have different possible values. Age,
height, IQ and so on are all variables since their values can change when applied to
different people. For example, Mr. X is a variable since X can represent anybody. A
variable may be qualitative or quantitative in nature.
These two types of variables are:
a) Quantitative variables: are variables that can be quantified or can have
numerical values. Examples: height, area, income, temperature e t c. Consider
the following questions; How many rooms are there in your house? Or How
many children are there in the family? Would be in numerical values.
b) Qualitative variables: are variables that cannot be quantified directly.
Examples: color, beauty, sex, location, political affiliation, and so on. Consider also
the following questions; “are you currently unemployed?” Would fit in the category
of either yes or no. Qualitative variables are also called categorical variables. And
hence we have two types of data; quantitative & qualitative data.
Quantitative variables can be further classified as
Discrete variables, and
Continuous variables
a) Discrete variables are variables whose values are counts. For examples:
• number of students attending a conference
• number of households (family size)
• Number of pages of a book
• number of eggs in the refrigerator, etc
b) Continuous variables are variables that can have any value within an interval.
5 
 
Examples:
• height of models in a beauty context
• weight of people joining a diet program
• Lengths of steel bars in a given production terms, e t c.
Scales of Measurement
Proper knowledge about the nature and type of data to be dealt with is essential in
order to specify and apply the proper statistical method for their analysis and
inferences. Measurement is the assignment of numbers to objects or events in a systematic 
fashion.  It is a functional mapping from the set of objects {Oi} to the set of real
numbers {M(Oi)}
Measurement scale refers to the property of value assigned to the data based on the
properties of order, distance and fixed zero. Four levels of measurement scales are
commonly distinguished: nominal, ordinal, interval, and ratio and each possessed
different properties of measurement systems 
Nominal scale: - “Nominal “is a Latin word for “name” which measures the presence
or absence of a characteristic of the data. The values of a nominal attribute are just
different names, i.e., nominal attributes provide only enough information to
distinguish one object from another. Qualities with no ranking or ordering; no
numerical or quantitative value. Data consists of names, labels and categories. This is
a scale for grouping individuals into different categories.
Examples: Car colors for a certain model are: red, silver, blue and black.
• In this scale, one is different from the other
• +, -, *, /, Impossible, comparison is impossible
Ordinal scale: - measures the presence or absence of a characteristic of the data using
an implied ranking or order. “Ordinal” is a Latin word, meaning “order”.
• Can be arranged in some order, but the differences between the data values are
meaningless.
• Data consisting of an ordering of ranking of measurements are said to be on an
ordinal scale of measurements. That is, the values of an ordinal scale provide
enough information to order objects. (<, >)
• One is different from and grater /better/ less than the other
• +, -, *, / Are impossible, comparison is possible.
Consider the following examples about ordinal scale:
a. Man A weighs more than man B
b. Ethiopian athletes got 1st
and 2nd
ranks in the 10,000m women’s final
in Beijing.
c. Of 17 fishing reels rated: 6 were rated good quality, 4 were rated better
quality, and 7 were rated best quality.
d. Out of a high school class of 319, Walter ranked 4th, June ranked
12th, and Jim ranked 20th.
6 
 
Interval Level: Data values can be ranked and the differences between data values
are meaningful. However, there is no intrinsic zero, or starting point, and the ratio of
data values are meaningless. Note: Celsius & Fahrenheit temperature readings have
no meaningful zero and ratios are meaningless.
In this measurement scale:-
• One is different, better/greater by a certain amount of difference than another
• Possible to add and subtract. For example; 37O
c –35o
c = 2o
c, 45o
c – 43 o
c= 2o
c
• Multiplication and division are not possible. For example; 40o
c = 2(20o
c) But
this does not imply that an object which is 40 o
c is twice as hot as an object
which is 20 o
c
Interval scale data convey better information than nominal and ordinal scale.
Most common examples are: IQ, temperature, Calendar dates, etc
Consider the following examples:
a. The years in which democrats won presidential elections.
b. Body temperature in degrees Celsius (or Fahrenheit)
c. Building A was built in 1284, Building B in 1492 and Building C in 5 B.C.
Ratio scale: Similar to interval, except there is a true zero, or starting point, and the
ratios of data values have meaning.
Best type of data: can add, subtract, multiply and divide. For ratio variables,
both differences and ratios are meaningful.
One is different, larger /taller/ better/ less by a certain amount of difference
and so much times than the other.
This measurement scale provides better information than interval scale of
measurement
Examples: temperature in Kelvin, monetary quantities, counts, age, mass, length,
electric current.
Consider the following some more examples:
a. Core temperatures of stars measures in degrees Kelvin
b. Time elapsed between the deposit of a check and the clearance of that check.
c. Length of the Nile River.
2. Methods of Data Collection and Presentation 
Methods of Data Collection
Data: are the values (measurements or observations) that the variables can assume.
Variables that are determined by chance are called random variables.
Any aggregate of numbers cannot be called statistical data. We say an aggregate of
numbers is statistical data when they are
• Comparable
• Meaningful and
• Collected for a well defined objective
Raw data: are collected data, which have not been organized numerically.
Examples: 25, 10, 32, 18, 6, 93, 4.
7 
 
The required data can be obtained from either a primary source or a secondary source.
Primary source: Is a source of data that supplies first hand information for the use of
the immediate purpose.
Primary Data: data you collect to answer your question. Data measured or collected
by the investigator or the user directly from the source. Or data originally collected
for the immediate purpose.
Two activities involved: planning and measuring.
a) Planning:
• Identify source and elements of the data.
• Decide whether to consider sample or census.
• If sampling is preferred, decide on sample size, selection method,… etc
• Decide measurement procedure.
• Set up the necessary organizational structure.
b) Measuring: there are different options.
• Focus Group
• Telephone Interview
• Mail Questionnaires
• Door-to-Door Survey
• Mall Intercept
• New Product Registration
• Personal Interview and
• Experiments are some of the sources for collecting the primary data.
Secondary source: are individuals or agencies, which supply data originally
collected for other purposes by them or others. Usually they are published or
unpublished materials, records, reports, e t c.
Secondary data: data collected from a secondary source by other people for other
purposes. Data gathered or compiled from published and unpublished sources or files.
When our source is secondary data check that:
• The type and objective of the situations.
• The purpose for which the data are collected and compatible with the present
problem.
• The nature and classification of data is appropriate to our problem.
• There are no biases and misreporting in the published data.
Note: Data which are primary for one may be secondary for the other.
Methods of Data Presentation
Having collected and edited the data, the next important step is to organize it. That is
to present it in a readily comprehensible condensed form that aids in order to draw
inferences from it. It is also necessary that the like be separated from the unlike ones.
The presentation of data is broadly classified in to the following two categories:
• Tabular presentation
• Diagrammatic and Graphic presentation.
The process of arranging data in to classes or categories according to similarities
technically is called classification. It eliminates inconsistency and also brings out the
8 
 
points of similarity and/or dissimilarity of collected items/data. It is necessary because
it would not be possible to draw inferences and conclusions if we have a large set of
collected [raw] data.
Frequency Distributions
Frequency: - is the number of times a certain value or set of values occurs in a
specific group.
A frequency distribution is a table that presents data according to some criteria with
the corresponding number of items falling in each class (i.e. with the corresponding
frequencies.). We see at a glance the shape of the distribution, the range of variation,
and any clustering of the values. By presenting a frequency distribution in relative
form, i.e., as percentages, we convert to the familiar base of 100 and make it easy to
compare the distribution of cases between different variables and/or different samples,
each of which may involve different total numbers of cases.
Generally, there are three basic types of frequency distributions: Categorical,
Ungrouped and Grouped frequency distributions.
1. Categorical frequency distribution
– the data are usually qualitative
– the scales of measurements for the data are usually nominal or ordinal
For instance data on blood types of people, political affiliation, economic status (low,
medium and high), religious affiliation are presented by categorical frequency
distributions.
Example: Thirty students, last year, took Stat 100 course and their grades were as
follows. Construct an appropriate frequency distribution for these data.
Table 2.1: Grades of students
B B C B A C
D C C C B B
B A B C D C
A F B F C A
B C C A C D
Solution:
There are five kinds of grades: A, B, C, D and F which may be used as the classes for
constructing the distribution. The procedure for constructing a frequency distribution
for categorical data is given below.
STEP 1. Construct a table as shown below
STEP 2. Tally the data and place the results
STEP 3. Count the Tallies and put the results
9 
 
STEP 4. Calculate the percentages (%) of frequencies in each class ( ) 100% ×=
n
f
Where = frequency of the class = total number of observationsf n
Class
( I )
Tally
( II )
Frequency
( III )
Percent*
( IV )
A ///// 5 16.7
B ///// //// 9 30.0
C ///// ///// / 11 36.7
D /// 3 10.0
F // 2 6.7
2. Ungrouped frequency distribution
Ungrouped frequency distribution is a table of all potential raw scored values that
could possibly occur in the data along with their corresponding frequencies. It is often
constructed for small set of data or a discrete variable.
Constructing an ungrouped frequency distribution
To construct an ungrouped frequency distribution, first find the smallest and the
largest raw scores in the collected data. Then make a columnar table of all potential
raw scored values arranged in order of magnitude with the number of times a
particular value is repeated, i.e., the frequency of that value. To facilitate counting
method, tallies can be used.
Example: The following data are the ages in years of 20 women who attend health
education last year: 30, 41, 39, 41, 32, 29, 35, 31, 30, 36, 33, 36, 32, 42, 30, 35, 37,
32, 30, and 41. Construct a frequency distribution for these data.
Solution:
STEP 1. Find the range: nobservatioMinimumnobservatioMaximumRange −=
STEP 2. Construct a table, tally the data and complete the frequency column. The
frequency distribution becomes as follows.
Age 29 30 31 32 33 35 36 37 39 41 42
Tally / //// / /// / // // / / /// /
Frequency 1 4 1 3 1 2 2 1 1 3 1
3. Grouped frequency distribution
STEP 1. Determine the unit of measurement, U
STEP 2. Find the maximum(Max) and the minimum(Min) observation, and then
compute their range, R MinMaxRange −=
STEP 3. Fix the number of classes’ desired (k). there are two ways to fix k:
• Fix k arbitrarily between 6 and 20, or
• Use Sturge’s Formula: Nk 10log332.31+= where N is the total frequency.
And round this value of k up to get an integer number.
10 
 
STEP 4. Find the class widths (W) by dividing the range by the number of classes and
round the number up to get an integer value.
K
RW =
STEP 5. Pick a suitable starting point less than or equal to the minimum value. This
starting point is the lower limit of the first class. Continue to add the class
width to this lower limit to get the rest of the lower limits.
STEP 6. Find the upper class limits. To find the upper class limit of the first class, subtract
one unit of measurement from the lower limit of the second class. Then continue to
add the class width to this upper limit to get the rest of the upper limits.
STEP 7. Compute the class boundaries as: ULCLLCB 2
1
−= and UUCLUCB 2
1
+=
Where LCL = lower class limit, UCL= upper class limit, LCB= lower class
boundary and UCB= upper class boundary. The class boundaries are also half way
between the upper limit of one class and the lower limit of the next class.
STEP 8. Tally the data.
STEP 9. Find the frequencies.
STEP 10. (If necessary) Find the cumulative frequencies (more than and less than types).
Example: The number of hours 40 employees spends on their job for the last 7 days
Table: Number of hours employees spend
62 50 35 36 31 43 43 43
41 31 65 30 41 58 49 41
37 62 27 47 65 50 45 48
27 53 40 29 63 34 44 32
58 61 38 41 26 50 47 37
Construct a suitable frequency distribution for these data using 8 classes
Solution:
1. Unit of measurement; U= 1year
2. Max = 65, Min = 26 so that R = 65-26 = 39
3. It is already determined to construct a frequency distribution having 8 classes.
4. Class width 5875.4
5
39 ≈==W
5. Starting point = 26 = lower limit of the first class. And hence the lower class
limits become: 26 31 36 41 46 51 56 61
6. Upper limit of the first class = 31-1 = 30. And hence the upper class limits become
30 35 40 45 50 55 60 65
The lower and the upper class limits (Steps 5 and 6) can be written as follows.
Class limits 26-30 31-35 36-40 41-45 46-50 51-55 56-60 61-65
7. By subtracting 0.5 units of measurement from the lower class limits and by
adding 0.5 units of measurement to the upper class limits, we can get lower and
upper class boundaries as follows.
Class
Boundaries
25.5-
30.5
30.5-
35.5
35.5-
40.5
40.5-
45.5
45.5-
50.5
50.5-
55.5
55.5-
60.5
60.5-
65.5
STEPS 8, 9 and 10 are displayed in the following table.
11 
 
Class limits Class
boundaries
Tally frequ
ency
Cumulative
frequency
(less than
type)
Cumulative
frequency
(more than
type)
26 – 30 25.5 – 30.5 ///// 5 5 40
31 – 35 30.5 – 35.5 ///// 5 10 35
36 – 40 35.5– 40.5 ///// 5 15 30
41 – 45 40.5– 45.5 ///// //// 9 24 25
46 – 50 45.5– 50.5 ///// // 7 31 16
51 – 55 50.5– 55.5 / 1 32 9
56 – 60 55.5– 60.5 // 2 34 8
61 – 65 60.5– 65.5 ///// / 6 40 6
Diagrammatic presentation of data: Bar charts, Pie-chart, Cartograms
The most convenient and popular way of describing data is using graphical
presentation. It is easier to understand and interpret data when they are presented
graphically than using words or a frequency table. A graph can present data in a
simple and clear way. Also it can illustrate the important aspects of the data. This
leads to better analysis and presentation of the data.
What it is?
Graphs are pictorial representations of the relationships between two (or more)
variables and are an important part of descriptive statistics. Different types of graphs
can be used for illustration purposes depending on the type of variable (nominal,
ordinal, or interval) and the issues of interest.
When to Use It?
Graphs can be used any time one wants to visually summarize the relationships
between variables, especially if the data set is large or unmanageable. They are
routinely used in reports to underscore a particular statement about a data set and to
enhance readability. Graphs can appeal to visual memory in ways that mere tallies,
tables, or frequency distributions cannot. However, if not used carefully, graphs can
misrepresent relationships between variables or encourage inaccurate conclusions.
Importance:
• They have greater attraction.
• They facilitate comparison.
• They are easily understandable.
The three most commonly used diagrammatic presentation for discrete as well as
qualitative data are: • Pie charts, • Pictogram, • Bar charts
Pie chart
A pie chart is a circle that is divided in to sections or wedges according to the
percentage of frequencies in each category of the distribution. The angle of the sector
is obtained using:
360*
o
quantitywholethe
parttheofValue
SectorofAngle =
12 
 
Example
The following data are the blood types of 50 volunteers at a blood plasma donation
clinic: O A O AB A A O O B A O A AB B O O O A B A A O A A
O B A O AB A O O A B A A A O B O O A O A B O AB A O B
a) Organize this data using a categorical frequency distribution
b) Present the data using both a pie and a bar chart.
Solution
a) The classes of the frequency distribution are A, B, O, AB. Count the number of
donors for each of the blood types.
Table: the number of donors by blood types.
Blood
type Frequency Percent
A 19 38
B 8 16
O 19 38
AB 4 8
Total 50 100
b) Pie chart
Find the percentage of donors for each blood type. In order to find the angles of
the sector for each blood type, multiply the corresponding percentage by 3600
and divide by 100.
Figure: Pie-chart of the data on blood types of donors.
AB
O
B
A
Blood type
Pictogram
In these diagrams, we represent data by means of some picture symbols. We decide
abut a suitable picture to represent a definite number of units in which the variable is
measured
Example: The following table shows the orange production in a plantation from
production year 1990-1993. Represent the data by a pictogram.
Solution:
Table: Orange productions from 1990 to 1993
Production year 1990 1991 1992 1993
Amount (in kg) 3000 3850 3500 5000
13 
 
Figure: Pictogram of the data on Orange productions from 1990 to 1993.
Bar Charts
Bar graphs are commonly used to show the number or proportion of nominal or
ordinal data which possess a particular attribute. They depict the frequency of each
category of data points as a bar rising vertically from the horizontal axis. Bar graphs
most often represent the number of observations in a given category, such as the
number of people in a sample falling into a given income or ethnic group. They can
be used to show the proportion of such data points. Bar graphs are especially good for
showing how nominal data change over time. They are useful for comparing
aggregate over time space. Bars can be drawn either vertically or horizontally.
There are different types of bar charts. The most common being: Simple bar chart,
Component or sub divided bar chart, and Multiple bar charts.
Simple Bar Chart: Are used to display data on one variable and are thick lines
(narrow rectangles) having the same breadth.
Example: Suppose that the following were the gross revenues (in $100,000.00) for
company XYZ for the years 1989, 1990 and 1991. Draw the bar graph for this data.
Table: Gross revenue of companies
Year Revenue
1989 110
1990 95
1991 65
Solution:
The bar diagram for this data can be constructed as follows with the revenues
represented on the vertical axis and the years represented on the horizontal axis.
199119901989
110
100
90
80
70
60
Year
SumofRevenue
14 
 
Component Bar chart
When there is a desire to show how a total (or aggregate) is divided in to its
component parts, we use component bar chart. The bars represent total value of a
variable with each total broken in to its component parts and different colors or
designs are used for identifications
Example: Construct a sub-divided bar chart for the four types of products in relation
to the opinion of consumers purchasing the given products as given below:
Table: Opinion of consumers purchasing the given products
Products Definitely Probably Unsure No
Product 1 50% 40% 10% 2%
Product 2 60% 30% 12% 15%
Product 3 70% 45% 8% 8%
Product 4 60% 35% 5% 20%
Component Bar Chart
0%
50%
100%
1 2 3 4
Products
Opinionof
Consumers
NO
Unsure
Probably
Definitely
Multiple Bar charts
These are used to display data on more than one variable.
They are used for comparing different variables at the same time.
Example: Construct a sub-divided bar chart for the 3 types of expenditures in dollars
for a family of four for the years 1, 2, 3 and 4 as given below:
Table: Expenditures in dollars for families
Year Food Education Other Total
1 3000 2000 3000 8000
2 3500 3000 4000 10500
3 4000 3500 5000 12500
5000 5000 6000 16000
Bar Charts
0
2000
4000
6000
8000
1 2 3 4
Year
Typesof
expenditures
Food
Education
Other
Total
Figure: Multiple bar chart for Expenditures in dollars for families
15 
 
Graphical presentation of data: Histogram, and Frequency Polygon
The histogram, frequency polygon and cumulative frequency graph or ogives are most
commonly applied graphical representation for continuous data.
Procedures for constructing statistical graphs:
• Draw and label the X and Y axes.
• Choose a suitable scale for the (cumulative) frequencies and label it on the Y axes.
• Represent the class boundaries for the histogram or ogive or the mid points for the
frequency polygon on the X axes.
• Plot the points.
• Draw the bars or lines to connect the points.
i. Histogram
Histograms or column bar charts are common ways of presenting frequency in a
number of categories. Commonly used graphical presentation methods also include
the frequency polygon and ogive. Histograms portray an unequal width frequency
distribution table for further statistical use. A frequency distribution table is a
tabulation of the n measurements into mutually exclusive k classes showing the
number of observations in each. The bars appear in a histogram where the classes are
marked on the x axis and the class frequencies on the y axis. The histogram is
constructed by creating x-axis units of equal size and these should correspond to the
frequency table. Histogram usually shows the number of observations in a specific
range.
Example: Construct a histogram for the frequency distribution of the time spent by
the automobile workers.
Table: Time in minutes spent by automobile workers
Time (in minute) Class mark Number of workers
15.5- 21.5 18.5 3
21.5-27.5 24.5 6
27.5-33.5 30.5 8
33.5-39.5 36.5 4
39.5-45.5 42.5 3
45.5-51.5 48.5 1
0
2
4
6
8
10
18.5 24.5 30.5 36.5 42.5 48.5
Frequency
Time
Figure: Histogram of the data on the number of minutes spent by the automobile workers.
16 
 
Table: Data given to present a Histogram
Class
Boundaries
Class
Mark
Frequency Cumulative
Frequency (less
than type)
Cumulative
Frequency (more
than type)
5.5 – 11.5 8.5 2 2 20
11.5 – 17.5 14.5 2 4 18
17.5 – 23.5 20.5 7 11 16
23.5 – 29.5 26.5 4 15 9
29.5 – 35.5 32.5 3 18 5
35.5 – 41.5 38.5 2 20 2
ii. Frequency Polygon
A frequency polygon is a line graph drawn by taking the frequencies of the classes
along the vertical axis and their respective class marks along the horizontal axis. Then
join the cross points by a free hand curve.
Example: Construct a frequency polygon for the frequency distribution of the time
spent by the automobile workers that we have seen in example 2.9.
Figure: Number of minutes spent by the automobile workers.
iii. Cumulative Frequency Polygon (Ogive)
It is a graph obtained by plotting the cumulative frequencies of a distribution against
the boundaries used to form the cumulative frequencies.
Example: Construct an ogive for the time spent by the automobile workers.
17 
 
3. Measures of Central Tendency
Introduction
Measures of Central Tendency give us information about the location of the center
of the distribution of data values. A single value that describes the characteristics of
the entire mass of data is called measures of central tendency. We will discuss briefly
the three measures of central tendency: mean, median and mode in this unit.
Suppose we have a random sample of n values of some measurement X. The values
are X1
, X2
, . . ,Xn. We want to summarize the information contained in this sample as
regards an "average" level. An average is a numerical value that indicates the middle
point or central region of the raw data. Mathematically summarize data in order to
make appropriate comparisons.
Objectives of measures of central tendency
Objectives of measuring central tendency are:
• To get a single value that represent(describe) characteristics of the entire data
• To summarizing/reducing the volume of the data
• To facilitating comparison within one group or between groups of data
• To enable further statistical analysis
The Summation Notation (∑): Let a data set consists of a number of observations,
represents by where n denotes the number of observations in the data and
is the ith
observation. Then, is the sum of all the observation∑=i
ix
1
N
∑
xxx n.,..,, 21
ix
For instance a data set consisting of six measurements 21, 13, 54, 46, 32 and 37 is
represented by and where = 21, = 13, = 54, = 46, =
32 and = 37.
54321 ,,,, xxxxx 6x 1x 2x 3x 4x 5x
6x
Their sum becomes 21+13+59+46+32+37=208.∑=
=
6
1i
ix
Similarly =
22
2
2
1 ... nxxx +++ ∑=
n
i
ix1
2
Some Properties of the Summation Notation
1. = where c is a constant number.∑=
n
i
c
1
cn.
2. where is a constant number∑ ∑= =
=
n
i
n
i
ii xbxb
1 1
. b
3. where a and b are constant numbers∑= =
+=+
n
i
n
i
ii xbanbxa
1 1
.)(
4. ∑∑∑ ===
±=±
n
i
i
n
i
i
n
i
ii yxyx
111
)(
5. ∑∑∑ ===
≠
n
i
i
n
i
i
n
i
ii yxyx
111
18 
 
Important Characteristics of a Good Average: A typical average should posses the 
following:  
• It should be rigidly defined.
• It should be based on all observation under investigation.
• It should be as little as affected by extreme observations.
• It should be capable of further algebraic treatment.
• It should be as little as affected by fluctuations of sampling.
• It should be ease to calculate and simple to understand.
i. Mean
The mean of a sample is determined by summing up all the data values of the sample
and dividing this sum by the total number of data values.
There are four types of means which are suitable for a particular type of data. These
are
1. Arithmetic Mean
2. Geometric Mean
3. Harmonic Mean
Arithmetic Mean
Arithmetic mean is defined as the sum of the measurements of the items divided by
the total number of items. It is usually denoted by x.
Suppose are n observed values in a sample of size n from a population
of size N, n<N then the arithmetic mean of the sample, denoted by
xx n
..,, 21x
x is given by
n
x
n
x....xx
=
in21 ∑=
+++
X
If we take an entire population Mean is denoted by µ and is given by:
NN
= N21
=μ
xx....xx i∑+++
Example: Consider the samples given for the two routes as follows:
For route A, the sample values are: 38 41 34 46 36 30 37 36 32 40
For route B the sample values are: 28 32 36 30 31 37 36 80 32 34 31
Find the arithmetic mean for the two routes.
Solution:
For route A, the sample values were: 38 41 34 46 36 30 37 36 32 40.
0.37
10
370
10
40+32+36+37+30+36+46+34+14+38
=x == minutes
For route B the sample values were: 28 32 36 30 31 37 36 80 32 34 31.
0.37
11
407
11
3134+32+80+36+37+31+30+36+32+28
=x ==
+
minutes
19 
 
When the numbers occur with frequencies ,
respectively, then mean can be expressed in a more compact form as
, ,   , … ,   , ,   , … ,  
 
∑
∑
Example
Calculate the arithmetic mean of the sample of numbers of people living in twelve
houses in a street: 3 2 4 4 3 1 2 5 3 3 8 1.
25.3
12
39
12
183+3+5+2+1+3+4+4+2+3
=x ==
+ +
The information in the sentence above can be written in a table, as follows.
Value, x 1 2 3 4 5 8
Frequency, f 2 2 4 2 1 1
 
The formula for the arithmetic mean for data of this type is
∑
∑
In this case we have:
.25.3
12
39
12
8581242
112422
81514241
==
3222 +++++
=
+++++
+×+
=x
×+×+× × + ×
The mean numbers of people living in twelve houses in a street is 3.25.
Arithmetic Mean for Grouped Frequency Distribution
If data are given in the form of continuous frequency distribution, the sample mean is:
fff
mfmfmf
f
mf
k
kk
k
i
i
i
ii
x
+++
+++
==
∑
∑
=
=
...
...
21
2211
1
1
i
nf
i
i =∑=1
th
i
k
Where m is the class mark of the class; i = 1, 2, …, k;
th
fi is the frequency of the i class and k = the number of classes
k
Note that = the total number of observations.
Example: The following table gives the height (inches) of 100 students in a college.
Class Interval (CI) Frequency (f)
60 - 62 5
62 – 64 18
64 – 66 42
66 – 68 20
68 – 70 8
70 – 72 7
Total 100
Calculate the mean?
20 
 
Solution:
∑
∑
=
=
= k
i
i
k
i
ii
f
mf
x
1
1
100
1
==∑=
nf
k
i
i ,
∑
∑
=
=
= k
i
i
k
i
ii
f
mf
x
1
1 =
100
6558
= 65.58
The mean height of students is 65.58
Properties of the Arithmetic Mean
The sum of the deviations of the items from their arithmetic mean is zero. This
means, the algebraic sum of the deviations of a set of numbers from
their mean
nxxx .,..,, 21
x is zero. That is
The sum of the squares of the deviations of a set of observations from any
number, say A, is minimum when A= . That is, ∑   ∑
0)(
1
=−∑=
n
i
i xx
When a set of observations is divided into k groups and 1x the mean of 1n
servations of group 1,
is
ob 2x the mean of n bservations of group2, …,is 2 o kx is
the mean of observations of group k , then the combined mean ,denoted bykn x ,
of all observations taken together is given by
If the mean of isnxxx .,..,, 21 x , then
a) the mean of kxkxkx n ±±± .,..,, 21 will be kx ±
b) The mean of will benkxkxkx .,..,, 21 xk .
Example
Last year there were three sections taking Stat 200 course. At the end of the semester,
the three sections got average marks of 80, 83 and 76. There were 28, 32 and 35
students in each section respectively. Find the mean mark for the entire students.
Solution:
==
++
++
=
++
++
=
95
7556
353228
)76(35)83(32)80(28
321
332211
nnn
xnxnxn
x 79.54
Weighted Arithmetic Mean
In finding arithmetic mean, all items were assumed to be of equally importance (each
value in the data set has equal weight). When the observations have different weight,
we use weighted average. Weights are assigned to each item in proportion to its
relative importance.
21 
 
If represent values of the items and are the corresponding
weights, then the weighted mean,
kxxx .,..,, 21 kwww ,...,, 21
)( wx is given by
1 1 2 2 1
1 2
1
...
...
k
i i
k k i
w k
k
i
i
w x
w x w x w x
X
w w w
w
=
=
+ + +
= =
+ + +
∑
∑
Example: A student’s final mark in Mathematics, Physics, Chemistry and Biology
are respectively 82, 80, 90 and 70.If the respective credits received for these courses
are 3, 5, 3 and 1, determine the approximate average mark the student has got for one
course.
Solution
We use a weighted arithmetic mean, weight associated with each course being taken
as the number of credits received for the corresponding course.
Therefore 17.82
1353
)701()903()805()823(
=
+++
×+×+×+×
==
∑
∑
i
ii
w
w
xw
x
Average mark of the student for one course is approximately 82.
Geometric Mean
The geometric mean like arithmetic mean is calculated average. It used when
observed values are measured as ratios, percentages, proportions, indices or growth
rates.
The geometric mean, G.M. of a series of numbers is defined asxn
xx .,..,, 21
n
nxxxGM ..... 21
= ,
Properties of geometric mean
a. Its calculations are not as such easy.
b. It involves all observations during computation
c. It may not be defined even it a single observation is negative.
d. If the value of one observation is zero its values becomes zero.
Remark:
1) When the observed values have the corresponding frequenciesxn
xx .,..,, 21
f n
ff .,..,, 21 respectively, the geometric mean is obtained by
n
kxxx
fffGM k..... 21
21
=
2)
The above formula can also be used whenever the frequency distribution are
grouped (continuous), class marks of the class intervals are considered as im
n
kmmm
fffGM k..... 21
21
= Where ∑=
=
n
i
i
fn
1
Example: Compute the geometric mean of the following values: 2, 8, 6, 4, 10, 6, 8, 4
Solution:
22 
 
n
kxxx
fffGM k..... 21
21
= = 8
1221
108642 ...2.
8
106436162 xxxxGM =
8
737280= = 5.41
The geometric mean for the given data will be 5.41
Harmonic Mean
It is a suitable measure of central tendency when the data pertains to speed, rate and time.
The harmonic mean is defined as the reciprocal of the arithmetic mean of the reciprocal
of the individual observations. If
are n observations, then harmonic mean
can be represented by the following formula:
xn
xx .,..,, 21
xxx n
n
i
i
nn
HM
1
....
11
1
1
++
==
∑ =
If the data arranged in the form of frequency distribution
xfxf
ff
xf
f
kk
k
n
i
i
k
i i
i
HM
++
++
==
∑
∑
=
=
........
1
.....
1
11
1
1
1
Properties of harmonic mean
i. It is based on all observation in a distribution.
ii. Used when a situations where small weight is give for larger
observation and larger weight for smaller observation
iii. Difficult to calculate and understand
iv. Appropriate measure of central tendency in situations where data is in
ratio, speed or rate
Example: A motorist travels 480km in 3 days. She travels for 10 hours at rate of
48km/hr on 1st
day, for 12 hours at rate of 40km/hr on the 2nd
day and for 15 hours at rate
of 32km/hr on the 3rd
day. What is her average speed?
92.39
32
1
40
1
48
1
3
=
++
=HM
Note: Whenever the frequency distribution are grouped (continuous), class marks of the
class intervals are considered as
and the above formula can be used asim
∑ =
=
n
i
i
m
f
n
HM
1
i
where mi is the class mark of ith class.∑=
=
n
i
i
fn
1
Relations among different means
1. If all the observations are positive we have the relationship among the three
means given as: HMGMx ≥≥
23 
 
2. For two observations GMHMx =*
3. HMGMx == if all observation are positive and have equal magnitude.
ii. Median
The median of a set of items (numbers) arranged in order of magnitude (i.e. in an
array form) is the middle value or the arithmetic mean of the two middle values. We
shall denote the median of by x~ .nxxx .,..,, 21
Median for Ungrouped Frequency Distribution
We arrange the sample in ascending order of the variable of interest. Then the median
is the middle value (if the sample size n is odd) or the average of the two middle
values (if the sample size n is even).
For ungrouped data the median is obtained by
⎪
⎪
⎪
⎪
⎩
⎪
⎪
⎪
⎪
⎨
⎧
+
=
⎟
⎠
⎞
⎜
⎝
⎛
+⎟
⎠
⎞
⎜
⎝
⎛
⎟
⎠
⎞
⎜
⎝
⎛ +
evenisnitemsofnumbertheif
itemitem
oddisnitemsofnumbertheifitem
x
nn
n
thth
th
,,,
2
,,,
~
1
22
2
1
Example: Compute the medians for the following two sets of data
A: 38 41 34 46 36 30 37 36 32 40.
B: 28 32 36 30 31 37 36 80 32 34 31
Solution
For A, the sample times in ascending order are: 30 32 34 36 36 37 38 40 41
46.
The two middle values have been underlined, So the median time ( x~
) =
5.36
2
3736
=
+
minutes.
For B, the sample times in ascending order are: 28 30 31 31 32 32 34 36
36 37 80.
The middle value has been underlined. So the median time ( x~
) = 32 minutes.
The median is easy to calculate for small samples and is not affected by an "outlier" -
atypical value in the sample, e.g. the value 80 in the sample for route B. It is valid for
ranked (ordinal) as well as interval/ratio (numeric) data.
Median for Grouped Frequency Distribution
For grouped data the median, obtained by interpolation method, is given by
 
24 
 
Where lower class boundary of the median class=medL
=F Sum of frequencies of all class lower than the median class (in other words it
is the cumulative frequency immediately preceding the median class)
Frequency of the median class and=medf
Class width=W
The median class is the class with the smallest cumulative frequency greater than or equal
to
2
n .
Example: Calculate the median for the following frequency distribution.
Class Class
Boundary
Frequency Less than Cum.
Freq (CF)
0 – 5 5 5
5 – 10 8 13
10 – 15 10 23
15 – 20 8 31
20 – 25 5 36
Solution:
To obtain the median class we divide 36 by 2. That is, 18
2
36
2
==n
Thus the smallest cumulative frequency that contains 18 is 23. Hence the median class is
the 3rd
class which is 10 – 15.
Therefore median is
 
Where 9.5, 13 sum of frequencies of all class lower than the 3rd
class=medL =F
10, frequency of the 3rd
class and,=medf =W 6, class width
5.12
35.9
10
1318
65.9~
=
+=
⎟
⎠
⎞
⎜
⎝
⎛ −
+=x
iii. Mode
The mode or the modal value is the most frequently occurring score/observation in a
series and denoted by . A data set may not have a mode or may have more than one
mode. A distribution is called a bimodal distribution if it has two data values that
appear with the greatest frequency. If a distribution has more than two modes, then
the distribution is multimodal. If a distribution has no modes, then the distribution is
non-modal.
xˆ
Example: Consider the following data:
Data X: 3, 4, 6, 12, 31, 8, 9, 8 the Mode ( ) = 8xˆ
Data Y: 6, 8, 12, 13, 11, 12, 6 the Mode ( ) = 6 and 12xˆ
Data Z: 2, 6, 3, 5, 7, 8, 12, 11 No Mode
Note that in some samples there may be more than one mode or there may not be a
mode. The mode is not a suitable measure of central tendency in these cases.
25 
 
Mode for Grouped Frequency Distribution
For grouped data, the mode is found by the following formula:
WLx ⎟⎟
⎠
⎜⎜
⎝ Δ+Δ
+=
21
1
mod
ˆ
⎞⎛ Δ
Where lower class boundary of the modal class=modL
11 ff −=Δ , 22 ff −=Δ
=1f Frequency of the modal class
=1f Frequency of the class immediately preceding the modal class
=2f Frequency of the class immediately follows the modal class
=W the class width
The modal class is the class with the highest frequency in the distribution.
Example: Find the mode for the frequency distribution of the birth weight (in kilogram)
of 30 children given below.
Weight 1.9-2.3 2.3-2.7 2.7-3.1 3.1-3.5 3.5-3.9 3.9-4.3
No. of children 5 5 9 4 4 3
Solution:
2.7- 3.1 is the modal class since it has the highest frequency
and4591 =−=Δ 5492 =−=Δ 7.2mod =L
878.24.0*
54
4
7.2ˆ =⎟
⎠
⎞
⎜
⎝
⎛
+
+=x  
The Relationship of the Mean, Median and Mode
• In the case of symmetrical distribution; mean, median and mode coincide.
That is mean=median = mode. However, for a moderately asymmetrical
(non symmetrical) distribution, mean and mode lie on the two ends and
median lies between them and they have the following important empirical
relationship: Mean – Mode = 3(Mean - Median) ⇒ ( )xxxx ~3ˆ −=−
Example In a moderately asymmetrical distribution, the mean and the median are 20
and 25 respectively. What is the mode of the distribution?
Solution:
Mode = 3median – 2mean = 3(25) – 2(20) = 75 – 40
= 35, hence the mode of the distribution will be 35.
4. MEASURES OF VARIATION (DISPERSION)
Introduction and objectives of measuring variation
We have seen that averages are representatives of a frequency distribution. But they
fail to give a complete picture of the distribution. They do not tell anything about the
spread or dispersion of observations within the distribution. Suppose that we have the
distribution of yield (kg per plot) of two rice varieties from 5 plots each.
26 
 
Variety 1: 45 42 42 41 40 Variety 2: 54 48 42 33 30
The mean yield of both varieties is 42 kg. The mean yield of variety 1 is close to the
values in this variety. On the other hand, the mean yield of variety 2 is not close to the
values in variety 2. The mean doesn’t tell us how the observations are close to each
other. This example suggests that a measure of central tendency alone is not sufficient
to describe a frequency distribution. Therefore, we should have a measure of spreads
of observations. There are different measures of dispersion.
Objectives of measuring variation
• To describe dispersion (variability) in a data.
• To compare the spread in two or more distributions.
• To determine the reliability of an average.
Note: The desirable properties of good measures of variation are almost identical with
that of a good measure of central tendency.
Absolute and relative measures
Absolute measures of dispersion are expressed in the same unit of measurement in
which the original data are given. These values may be used to compare the variation
in two distributions provided that the variables are in the same units and of the same
average size. In case the two sets of data are expressed in different units, however,
such as quintals of sugar versus tones of sugarcane or if the average sizes are very
different such as manager’s salary versus worker’s salary, the absolute measures of
dispersion are not comparable. In such cases measures of relative dispersion should be
used. A measure of relative dispersion is the ratio of a measure of absolute dispersion
to an appropriate measure of central tendency. It is a unitless measure.
Types of Measures of Variation
i. The range and relative range
Range(R) is defined as the difference between the maximum and minimum
observations in a set of data.
Range Maximum value Minimum value= −
It is the crudest absolute measures of variation. It is widely used in the construction of
quality control charts and description of daily temperature.
Properties of range
It is affected by extreme values.
It does not take into account all observations.
It is easy to calculate and simple to understand.
It does not tell anything about the distribution of values in the set of data
relative to some measures of central tendency.
Relative range (RR) is defined as
. .
Range
RR
Max value Min value
=
+
ii. The mean deviation and coefficient of mean deviation
Mean deviation (MD) is the average of the absolute deviations taken from a central
value, generally the mean or median.
1 1
| | | |
n k
i i i
i i
X
X X X X f
MD
n n
= =
− −
= =
∑ ∑
, 1 1
| | | |
n k
i i
i i
X
iX X X X
MD
n n
= =
− −
= =
∑ ∑
%
% % f
27 
 
Example: Calculate the mean deviation about the median and about the mean of the
following scores of students in a certain test. 6,7,7,10,10
1
1.6
5 5
i
X
MD
n
=
= = = =
| |
| 6 8| | 7 8|*2 |10 8|*2 8
k
i iX X f−
− + − + −∑
1
| |
| 6 7 | | 7 7 |*2 |10 7 |*2 7
1.4
5 5
k
i i
i
X
X X f
MD
n
=
−
− + − + −
= = =
∑
%
%
=
Note: In case of grouped data, the mid-point of each class interval is treated as and we
can use the above formula. Besides, X X
MD MD≥ %
Properties of mean deviation
It is relatively simple to understand as compared to standard deviation.
Its computation is simple.
It is less affected by extreme values than standard deviation.
It is better than the range and QD since it is based on all observations.
It is not suitable for further statistical treatment.
iii. Variance, standard deviation and coefficient of variation
The variance is the average of the squares of the distance each value is from the mean.
The symbol for the population variance is σ2
. Let 1 2, ,..., Nx x x be the measurements on
N population units then, the population variance is given by the formula:
2
12 2
2 1 1
( )
N
iN N
i
i i
i i
x
x x
N
N N
μ
σ
=
= =
⎛ ⎞
⎜ ⎟
⎝ ⎠− −
= =
∑
∑ ∑
,
Where µ is population mean and N is population size
Let 1 2, ,..., nx x x be the measurements on n sample units then, the sample variance is
denoted by S2
, and its formula is
2
12 2
2 1 1
( )
1 1
n
in n
i
i i
i i
x
x x x
n
S
n n
=
= =
⎛ ⎞
⎜ ⎟
⎝ ⎠− −
= =
− −
∑
∑ ∑
Where x is the sample mean and n is the sample size.
Standard deviation, denoted by σ or S, is the square root of the variance. That is,
Population standard deviation 2
σ σ= and sample standard deviation 2
S S=
.
Example: For a newly created position, a manager interviewed the following
numbers of applicants each day over a five-day period: 16, 19, 15, 15, and 14. Find
the variance and standard deviation.
Solution:
5
1 16 19 15 15 14 79
15.8
5 5 5
i
i
x
x = + + + +
= = = =
∑
28 
 
5
2
2 2
2 1
( )
(16 15.8) ... (14 15.8) 14.8
3.7
5 1 4 4
i
i
x x
S =
−
− + + −
⇒ = = = =
−
∑
Or
( )
2
25
12 2 2
2 1
16 ... 14 6241
16 ... 14 1263
14.85 5 5 3.7
5 1 4 4 4
n
i
i
i
i
x
x
S
=
=
⎛ ⎞
⎜ ⎟
+ +⎝ ⎠− + + − −
= = = =
−
∑
∑
=
For grouped frequency distribution, the formula for variance is
2
12 2
2 1 1
( )
1 1
k
i ik k
i
i i i i
i i
f x
f x x f x
n
S
n n
=
= =
⎛ ⎞
⎜ ⎟
⎝ ⎠− −
= =
− −
∑
∑ ∑
Where is the number of classes,k ix is the class mark of class i and
1
k
i
i
n f
=
= ∑
Properties of variance
The unit of measurement of the variance is the square of the unit of
measurement of the observed values. It is one of its limitations.
The variance gives more weight to extreme values as compared to those which
are near to the mean value, because the difference is squared in variance.
It is based on all observations in the data set.
Properties of standard deviation
Standard deviation is considered to be the best measure of dispersion and is
used widely.
There is, however, one difficulty with it. If the unit of measurement of
variables of two series is not the same, then their variability cannot be
compared by comparing the values of standard deviation.
Uses of the variance and standard deviation
The variance and standard deviations can be used to determine the spread of
data, consistency of a variable and the proportion of data values that fall
within a specified interval in a distribution.
If the variance or standard deviation is large, the data is more dispersed. This
information is useful in comparing two or more data sets to determine which is
more (most) variable.
Finally, the variance and standard deviation are used quite often in inferential
statistics.
Coefficient of variation (CV)
The standard deviation is an absolute measure of dispersion. The corresponding
relative measure is known as the coefficient of variation (CV).
Coefficient of variation is used in such problems where we want to compare the
variability of two or more different series. Coefficient of variation is the ratio of the
standard deviation to the arithmetic mean, usually expressed in percent:
*100%
s
CV
x
=
A distribution having less coefficient of variation is said to be less variable or more
consistent or more uniform or more homogeneous.
29 
 
Example: Last semester, the students of two departments, A and B took Stat 276
course. At the end of the semester, the following information was recorded.
Dept A Dept B
Mean score 79 64
SD 23 11
Compare the relative dispersion of the two departments.
23
*100% *100 29.11%
79
A
A
A
s
CV
x
= = = ,
11
*100% *100 17.19%
64
B
B
B
s
CV
x
= = =
Since ACV > , the variation is department A is greater. Or, in department B the
distribution of the marks is more uniform (consistent).
BCV
Exercise: The mean weight of 20 children was found to be 30 kg with variance of
16kg2
and their mean height was 150 cm with variance of 25cm2
. Compare the
variability of weight and height of these children.
iv. The standard scores
A standard score is a measure that describes the relative position of a single score in
the entire distribution of scores in terms of the mean and standard deviation. It also
gives us the number of standard deviations a particular observation lie above or below
the mean.
Standard score, ,
x x X
Z or Z
S
μ
σ
− −
= =
Where x is the value of the observation, / Xμ and / Sσ are the mean and standard
deviation of the respectively.
Interpretation:
,
,
,
Negative the observation lies below the mean
If Z is Positive theobservation lies above the mean
Zero the observationequals to the mean
⎧
⎪
⎨
⎪
⎩
Example: Two sections were given an exam in a course. The average score was 72
with standard deviation of 6 for section 1 and 85 with standard deviation of 5 for
section 2. Student A from section 1 scored 84 and student B from section 2 scored 90.
Who performed better relative to his/her group?
Solution:
Section 1: x = 72, S = 6 and score of student A from Section 1; A x = 84
Section 2: x = 85, S = 5 and score of student B from Section 2; B x = 90
Z-score of student A:
84 72
2
6
A A
A
A
x X
Z
S
− −
= = =
Z-score of student B:
90 85
1
5
B B
B
B
x X
Z
S
− −
= = =
From these two standard scores, we can conclude that student A has performed better
relative to his/her section students because his/her score is two standard deviations
above the mean score of selection 1 while the score of student B is only one standard
deviation above the mean score of section 2 students.
Exercise: A student scored 65 on a calculus test that had a mean of 50 and a standard
deviation of 10; she scored 30 on a algebra test with a mean of 25 and a standard
deviation of 5. Compare her relative positions on each test.
30 
 
v. Skewness and kurtosis
Skweness refers to lack of symmetry in a distribution. If a distribution is not
symmetrical we call it skewed distribution. Note that for a symmetrical and
unimodal distribution: Mean = median = mode
Measure of skewness:
3
ˆx x
s
α
−
Pearsonian coefficient of skewness (Pcsk) defined as: =
Interpretation: 3
0,
0,
0,
the distribution isnegatively skewed
If the distribution is positively skewed
the distribution is symetrical
α
<⎧
⎪
>⎨
⎪=⎩
In moderately skewed distributions: Mode = mean- 3(mean-median) 3
3( )x x
s
α
−
⇒ =
%
Note: in a negatively skewed distribution larger values are more frequent than smaller
values. In a positively skewed distribution smaller values are more frequent than
larger values.
Exercise: If the mean, mode and s.d of a frequency distribution are 70.2, 73.6, and
6.4, respectively. What can one state about its skeweness?
Kurtosis refers to the degree of peakedness of a distribution. When the values of a
distribution are closely bunched around the mode in such a way that the peak of
the distribution becomes relatively high, the distribution is said to be leptokurtic.
If it is flat topped we call it platykurtic. A distribution which is neither highly
peaked nor flat topped is known as a mesokurtic distribution (normal).
31 
 

More Related Content

What's hot

33 renal sinus abnormalities
33 renal sinus abnormalities33 renal sinus abnormalities
33 renal sinus abnormalities
Dr. Muhammad Bin Zulfiqar
 
Stem and leaf plot
Stem and leaf plotStem and leaf plot
Stem and leaf plot
Susan Vo
 
Demography
DemographyDemography
Demography
migom doley
 
Four data types Data Scientist should know
Four data types Data Scientist should knowFour data types Data Scientist should know
Four data types Data Scientist should know
Ranjit Nambisan
 
Population and Demography
Population and DemographyPopulation and Demography
Population and Demography
Ryan Cloyd Villanueva
 
Introduction To Statistics
Introduction To StatisticsIntroduction To Statistics
Introduction To Statisticsalbertlaporte
 
Demography
DemographyDemography
Demography
Marwa Salem
 
Population projection (30 1-2017) by dr min ko ko
Population projection (30 1-2017)  by dr min ko koPopulation projection (30 1-2017)  by dr min ko ko
Population projection (30 1-2017) by dr min ko ko
Mmedsc Hahm
 
Attributable risk and population attributable risk
Attributable risk and population attributable riskAttributable risk and population attributable risk
Attributable risk and population attributable risk
Abino David
 
Measures of Mortality
Measures of MortalityMeasures of Mortality
Measures of Mortality
Akhilesh Bhargava
 
Measures Of Association
Measures Of AssociationMeasures Of Association
Measures Of Association
ganesh kumar
 
Chap02 describing data; numerical
Chap02 describing data; numericalChap02 describing data; numerical
Chap02 describing data; numerical
Judianto Nugroho
 
Lec4 morbidity(revised07)
Lec4 morbidity(revised07)Lec4 morbidity(revised07)
Lec4 morbidity(revised07)
Abeer Esmail
 
What does an odds ratio or relative risk mean?
What does an odds ratio or relative risk mean? What does an odds ratio or relative risk mean?
What does an odds ratio or relative risk mean?
Terry Shaneyfelt
 
Basic concepts of statistics
Basic concepts of statistics Basic concepts of statistics
Basic concepts of statistics
Dr. Trilok Kumar Jain
 
44 DAVID SUTTON PICTURES SKELETAL TRAUMA REGIONAL
44 DAVID SUTTON PICTURES SKELETAL TRAUMA REGIONAL44 DAVID SUTTON PICTURES SKELETAL TRAUMA REGIONAL
44 DAVID SUTTON PICTURES SKELETAL TRAUMA REGIONAL
Dr. Muhammad Bin Zulfiqar
 
Skeletal disorders of metabolic origin
Skeletal disorders of metabolic originSkeletal disorders of metabolic origin
Skeletal disorders of metabolic origin
Navni Garg
 
Cumulative frequency distribution
Cumulative frequency distributionCumulative frequency distribution
Cumulative frequency distribution
MahrukhShehzadi1
 
Population Theorists
Population TheoristsPopulation Theorists
Population Theorists
James Foster
 

What's hot (20)

33 renal sinus abnormalities
33 renal sinus abnormalities33 renal sinus abnormalities
33 renal sinus abnormalities
 
Stem and leaf plot
Stem and leaf plotStem and leaf plot
Stem and leaf plot
 
Demography
DemographyDemography
Demography
 
Four data types Data Scientist should know
Four data types Data Scientist should knowFour data types Data Scientist should know
Four data types Data Scientist should know
 
Population and Demography
Population and DemographyPopulation and Demography
Population and Demography
 
Introduction To Statistics
Introduction To StatisticsIntroduction To Statistics
Introduction To Statistics
 
Demography
DemographyDemography
Demography
 
Population projection (30 1-2017) by dr min ko ko
Population projection (30 1-2017)  by dr min ko koPopulation projection (30 1-2017)  by dr min ko ko
Population projection (30 1-2017) by dr min ko ko
 
Attributable risk and population attributable risk
Attributable risk and population attributable riskAttributable risk and population attributable risk
Attributable risk and population attributable risk
 
Measures of Mortality
Measures of MortalityMeasures of Mortality
Measures of Mortality
 
Measures Of Association
Measures Of AssociationMeasures Of Association
Measures Of Association
 
Chap02 describing data; numerical
Chap02 describing data; numericalChap02 describing data; numerical
Chap02 describing data; numerical
 
Lec4 morbidity(revised07)
Lec4 morbidity(revised07)Lec4 morbidity(revised07)
Lec4 morbidity(revised07)
 
What does an odds ratio or relative risk mean?
What does an odds ratio or relative risk mean? What does an odds ratio or relative risk mean?
What does an odds ratio or relative risk mean?
 
Basic concepts of statistics
Basic concepts of statistics Basic concepts of statistics
Basic concepts of statistics
 
Variability
VariabilityVariability
Variability
 
44 DAVID SUTTON PICTURES SKELETAL TRAUMA REGIONAL
44 DAVID SUTTON PICTURES SKELETAL TRAUMA REGIONAL44 DAVID SUTTON PICTURES SKELETAL TRAUMA REGIONAL
44 DAVID SUTTON PICTURES SKELETAL TRAUMA REGIONAL
 
Skeletal disorders of metabolic origin
Skeletal disorders of metabolic originSkeletal disorders of metabolic origin
Skeletal disorders of metabolic origin
 
Cumulative frequency distribution
Cumulative frequency distributionCumulative frequency distribution
Cumulative frequency distribution
 
Population Theorists
Population TheoristsPopulation Theorists
Population Theorists
 

Similar to Basic stat

Bahir dar institute of technology.pdf
Bahir dar institute of technology.pdfBahir dar institute of technology.pdf
Bahir dar institute of technology.pdf
Hailsh
 
INTRODUCTION TO STATISTICS.pptx
INTRODUCTION TO STATISTICS.pptxINTRODUCTION TO STATISTICS.pptx
INTRODUCTION TO STATISTICS.pptx
Manish Agarwal
 
Stats notes
Stats notesStats notes
Stats notes
Prabal Chakraborty
 
Statistics / Quantitative Techniques Study Material
Statistics / Quantitative Techniques Study MaterialStatistics / Quantitative Techniques Study Material
Statistics / Quantitative Techniques Study Material
Prabal Chakraborty
 
Statistics and its application
Statistics and its applicationStatistics and its application
Statistics and its application
gopinathannsriramachandraeduin
 
Statistics...
Statistics...Statistics...
probability and statistics Chapter 1 (1)
probability and statistics Chapter 1 (1)probability and statistics Chapter 1 (1)
probability and statistics Chapter 1 (1)abfisho
 
Introduction to Business Statistics
Introduction to Business StatisticsIntroduction to Business Statistics
Introduction to Business Statistics
SOMASUNDARAM T
 
Statistics Exericse 29
Statistics Exericse 29Statistics Exericse 29
Statistics Exericse 29
Melanie Erickson
 
Chapter 1 Introduction to statistics, Definitions, scope and limitations.pptx
Chapter 1 Introduction to statistics, Definitions, scope and limitations.pptxChapter 1 Introduction to statistics, Definitions, scope and limitations.pptx
Chapter 1 Introduction to statistics, Definitions, scope and limitations.pptx
SubashYadav14
 
Business Statistics.pdf
Business Statistics.pdfBusiness Statistics.pdf
Business Statistics.pdf
ssuser25bd39
 
Business Statistics.pdf
Business Statistics.pdfBusiness Statistics.pdf
Business Statistics.pdf
sivakumar740407
 
CHAPONE edited Stat.pptx
CHAPONE edited Stat.pptxCHAPONE edited Stat.pptx
CHAPONE edited Stat.pptx
BereketDesalegn5
 
Statistics for Managers notes.pdf
Statistics for Managers notes.pdfStatistics for Managers notes.pdf
Statistics for Managers notes.pdf
Velujv
 
Statistics assignment
Statistics assignmentStatistics assignment
Statistics assignment
Pragati Mehndiratta
 
Definition Of Statistics
Definition Of StatisticsDefinition Of Statistics
Definition Of Statistics
Joshua Rumagit
 
INTRO to STATISTICAL THEORY.pdf
INTRO to STATISTICAL THEORY.pdfINTRO to STATISTICAL THEORY.pdf
INTRO to STATISTICAL THEORY.pdf
mt6280255
 
probability and statistics-4.pdf
probability and statistics-4.pdfprobability and statistics-4.pdf
probability and statistics-4.pdf
habtamu292245
 
Basics of Research Types of Data Classification
Basics of Research Types of Data ClassificationBasics of Research Types of Data Classification
Basics of Research Types of Data Classification
Harshit Pandey
 

Similar to Basic stat (20)

Bahir dar institute of technology.pdf
Bahir dar institute of technology.pdfBahir dar institute of technology.pdf
Bahir dar institute of technology.pdf
 
INTRODUCTION TO STATISTICS.pptx
INTRODUCTION TO STATISTICS.pptxINTRODUCTION TO STATISTICS.pptx
INTRODUCTION TO STATISTICS.pptx
 
Stats notes
Stats notesStats notes
Stats notes
 
Statistics / Quantitative Techniques Study Material
Statistics / Quantitative Techniques Study MaterialStatistics / Quantitative Techniques Study Material
Statistics / Quantitative Techniques Study Material
 
Overview of statistics
Overview of statisticsOverview of statistics
Overview of statistics
 
Statistics and its application
Statistics and its applicationStatistics and its application
Statistics and its application
 
Statistics...
Statistics...Statistics...
Statistics...
 
probability and statistics Chapter 1 (1)
probability and statistics Chapter 1 (1)probability and statistics Chapter 1 (1)
probability and statistics Chapter 1 (1)
 
Introduction to Business Statistics
Introduction to Business StatisticsIntroduction to Business Statistics
Introduction to Business Statistics
 
Statistics Exericse 29
Statistics Exericse 29Statistics Exericse 29
Statistics Exericse 29
 
Chapter 1 Introduction to statistics, Definitions, scope and limitations.pptx
Chapter 1 Introduction to statistics, Definitions, scope and limitations.pptxChapter 1 Introduction to statistics, Definitions, scope and limitations.pptx
Chapter 1 Introduction to statistics, Definitions, scope and limitations.pptx
 
Business Statistics.pdf
Business Statistics.pdfBusiness Statistics.pdf
Business Statistics.pdf
 
Business Statistics.pdf
Business Statistics.pdfBusiness Statistics.pdf
Business Statistics.pdf
 
CHAPONE edited Stat.pptx
CHAPONE edited Stat.pptxCHAPONE edited Stat.pptx
CHAPONE edited Stat.pptx
 
Statistics for Managers notes.pdf
Statistics for Managers notes.pdfStatistics for Managers notes.pdf
Statistics for Managers notes.pdf
 
Statistics assignment
Statistics assignmentStatistics assignment
Statistics assignment
 
Definition Of Statistics
Definition Of StatisticsDefinition Of Statistics
Definition Of Statistics
 
INTRO to STATISTICAL THEORY.pdf
INTRO to STATISTICAL THEORY.pdfINTRO to STATISTICAL THEORY.pdf
INTRO to STATISTICAL THEORY.pdf
 
probability and statistics-4.pdf
probability and statistics-4.pdfprobability and statistics-4.pdf
probability and statistics-4.pdf
 
Basics of Research Types of Data Classification
Basics of Research Types of Data ClassificationBasics of Research Types of Data Classification
Basics of Research Types of Data Classification
 

More from kula jilo

Clinical Pathology Laboratory Report by Kula Jilo 2016
Clinical Pathology Laboratory Report by Kula Jilo 2016Clinical Pathology Laboratory Report by Kula Jilo 2016
Clinical Pathology Laboratory Report by Kula Jilo 2016
kula jilo
 
BORANA OROMO CALENDER
BORANA  OROMO CALENDERBORANA  OROMO CALENDER
BORANA OROMO CALENDER
kula jilo
 
Kula jilo
Kula jiloKula jilo
Kula jilo
kula jilo
 
Booruu
BooruuBooruu
Booruu
kula jilo
 
Patho word
Patho wordPatho word
Patho word
kula jilo
 
Dhalootnii kun
Dhalootnii kunDhalootnii kun
Dhalootnii kun
kula jilo
 
2.04.02 bovine babesiosis
2.04.02 bovine babesiosis2.04.02 bovine babesiosis
2.04.02 bovine babesiosis
kula jilo
 

More from kula jilo (7)

Clinical Pathology Laboratory Report by Kula Jilo 2016
Clinical Pathology Laboratory Report by Kula Jilo 2016Clinical Pathology Laboratory Report by Kula Jilo 2016
Clinical Pathology Laboratory Report by Kula Jilo 2016
 
BORANA OROMO CALENDER
BORANA  OROMO CALENDERBORANA  OROMO CALENDER
BORANA OROMO CALENDER
 
Kula jilo
Kula jiloKula jilo
Kula jilo
 
Booruu
BooruuBooruu
Booruu
 
Patho word
Patho wordPatho word
Patho word
 
Dhalootnii kun
Dhalootnii kunDhalootnii kun
Dhalootnii kun
 
2.04.02 bovine babesiosis
2.04.02 bovine babesiosis2.04.02 bovine babesiosis
2.04.02 bovine babesiosis
 

Recently uploaded

The Roman Empire A Historical Colossus.pdf
The Roman Empire A Historical Colossus.pdfThe Roman Empire A Historical Colossus.pdf
The Roman Empire A Historical Colossus.pdf
kaushalkr1407
 
Mule 4.6 & Java 17 Upgrade | MuleSoft Mysore Meetup #46
Mule 4.6 & Java 17 Upgrade | MuleSoft Mysore Meetup #46Mule 4.6 & Java 17 Upgrade | MuleSoft Mysore Meetup #46
Mule 4.6 & Java 17 Upgrade | MuleSoft Mysore Meetup #46
MysoreMuleSoftMeetup
 
A Strategic Approach: GenAI in Education
A Strategic Approach: GenAI in EducationA Strategic Approach: GenAI in Education
A Strategic Approach: GenAI in Education
Peter Windle
 
Guidance_and_Counselling.pdf B.Ed. 4th Semester
Guidance_and_Counselling.pdf B.Ed. 4th SemesterGuidance_and_Counselling.pdf B.Ed. 4th Semester
Guidance_and_Counselling.pdf B.Ed. 4th Semester
Atul Kumar Singh
 
Model Attribute Check Company Auto Property
Model Attribute  Check Company Auto PropertyModel Attribute  Check Company Auto Property
Model Attribute Check Company Auto Property
Celine George
 
TESDA TM1 REVIEWER FOR NATIONAL ASSESSMENT WRITTEN AND ORAL QUESTIONS WITH A...
TESDA TM1 REVIEWER  FOR NATIONAL ASSESSMENT WRITTEN AND ORAL QUESTIONS WITH A...TESDA TM1 REVIEWER  FOR NATIONAL ASSESSMENT WRITTEN AND ORAL QUESTIONS WITH A...
TESDA TM1 REVIEWER FOR NATIONAL ASSESSMENT WRITTEN AND ORAL QUESTIONS WITH A...
EugeneSaldivar
 
The Challenger.pdf DNHS Official Publication
The Challenger.pdf DNHS Official PublicationThe Challenger.pdf DNHS Official Publication
The Challenger.pdf DNHS Official Publication
Delapenabediema
 
Operation Blue Star - Saka Neela Tara
Operation Blue Star   -  Saka Neela TaraOperation Blue Star   -  Saka Neela Tara
Operation Blue Star - Saka Neela Tara
Balvir Singh
 
Introduction to AI for Nonprofits with Tapp Network
Introduction to AI for Nonprofits with Tapp NetworkIntroduction to AI for Nonprofits with Tapp Network
Introduction to AI for Nonprofits with Tapp Network
TechSoup
 
Biological Screening of Herbal Drugs in detailed.
Biological Screening of Herbal Drugs in detailed.Biological Screening of Herbal Drugs in detailed.
Biological Screening of Herbal Drugs in detailed.
Ashokrao Mane college of Pharmacy Peth-Vadgaon
 
Additional Benefits for Employee Website.pdf
Additional Benefits for Employee Website.pdfAdditional Benefits for Employee Website.pdf
Additional Benefits for Employee Website.pdf
joachimlavalley1
 
Thesis Statement for students diagnonsed withADHD.ppt
Thesis Statement for students diagnonsed withADHD.pptThesis Statement for students diagnonsed withADHD.ppt
Thesis Statement for students diagnonsed withADHD.ppt
EverAndrsGuerraGuerr
 
1.4 modern child centered education - mahatma gandhi-2.pptx
1.4 modern child centered education - mahatma gandhi-2.pptx1.4 modern child centered education - mahatma gandhi-2.pptx
1.4 modern child centered education - mahatma gandhi-2.pptx
JosvitaDsouza2
 
Supporting (UKRI) OA monographs at Salford.pptx
Supporting (UKRI) OA monographs at Salford.pptxSupporting (UKRI) OA monographs at Salford.pptx
Supporting (UKRI) OA monographs at Salford.pptx
Jisc
 
How to Make a Field invisible in Odoo 17
How to Make a Field invisible in Odoo 17How to Make a Field invisible in Odoo 17
How to Make a Field invisible in Odoo 17
Celine George
 
The geography of Taylor Swift - some ideas
The geography of Taylor Swift - some ideasThe geography of Taylor Swift - some ideas
The geography of Taylor Swift - some ideas
GeoBlogs
 
Overview on Edible Vaccine: Pros & Cons with Mechanism
Overview on Edible Vaccine: Pros & Cons with MechanismOverview on Edible Vaccine: Pros & Cons with Mechanism
Overview on Edible Vaccine: Pros & Cons with Mechanism
DeeptiGupta154
 
CLASS 11 CBSE B.St Project AIDS TO TRADE - INSURANCE
CLASS 11 CBSE B.St Project AIDS TO TRADE - INSURANCECLASS 11 CBSE B.St Project AIDS TO TRADE - INSURANCE
CLASS 11 CBSE B.St Project AIDS TO TRADE - INSURANCE
BhavyaRajput3
 
The French Revolution Class 9 Study Material pdf free download
The French Revolution Class 9 Study Material pdf free downloadThe French Revolution Class 9 Study Material pdf free download
The French Revolution Class 9 Study Material pdf free download
Vivekanand Anglo Vedic Academy
 
Home assignment II on Spectroscopy 2024 Answers.pdf
Home assignment II on Spectroscopy 2024 Answers.pdfHome assignment II on Spectroscopy 2024 Answers.pdf
Home assignment II on Spectroscopy 2024 Answers.pdf
Tamralipta Mahavidyalaya
 

Recently uploaded (20)

The Roman Empire A Historical Colossus.pdf
The Roman Empire A Historical Colossus.pdfThe Roman Empire A Historical Colossus.pdf
The Roman Empire A Historical Colossus.pdf
 
Mule 4.6 & Java 17 Upgrade | MuleSoft Mysore Meetup #46
Mule 4.6 & Java 17 Upgrade | MuleSoft Mysore Meetup #46Mule 4.6 & Java 17 Upgrade | MuleSoft Mysore Meetup #46
Mule 4.6 & Java 17 Upgrade | MuleSoft Mysore Meetup #46
 
A Strategic Approach: GenAI in Education
A Strategic Approach: GenAI in EducationA Strategic Approach: GenAI in Education
A Strategic Approach: GenAI in Education
 
Guidance_and_Counselling.pdf B.Ed. 4th Semester
Guidance_and_Counselling.pdf B.Ed. 4th SemesterGuidance_and_Counselling.pdf B.Ed. 4th Semester
Guidance_and_Counselling.pdf B.Ed. 4th Semester
 
Model Attribute Check Company Auto Property
Model Attribute  Check Company Auto PropertyModel Attribute  Check Company Auto Property
Model Attribute Check Company Auto Property
 
TESDA TM1 REVIEWER FOR NATIONAL ASSESSMENT WRITTEN AND ORAL QUESTIONS WITH A...
TESDA TM1 REVIEWER  FOR NATIONAL ASSESSMENT WRITTEN AND ORAL QUESTIONS WITH A...TESDA TM1 REVIEWER  FOR NATIONAL ASSESSMENT WRITTEN AND ORAL QUESTIONS WITH A...
TESDA TM1 REVIEWER FOR NATIONAL ASSESSMENT WRITTEN AND ORAL QUESTIONS WITH A...
 
The Challenger.pdf DNHS Official Publication
The Challenger.pdf DNHS Official PublicationThe Challenger.pdf DNHS Official Publication
The Challenger.pdf DNHS Official Publication
 
Operation Blue Star - Saka Neela Tara
Operation Blue Star   -  Saka Neela TaraOperation Blue Star   -  Saka Neela Tara
Operation Blue Star - Saka Neela Tara
 
Introduction to AI for Nonprofits with Tapp Network
Introduction to AI for Nonprofits with Tapp NetworkIntroduction to AI for Nonprofits with Tapp Network
Introduction to AI for Nonprofits with Tapp Network
 
Biological Screening of Herbal Drugs in detailed.
Biological Screening of Herbal Drugs in detailed.Biological Screening of Herbal Drugs in detailed.
Biological Screening of Herbal Drugs in detailed.
 
Additional Benefits for Employee Website.pdf
Additional Benefits for Employee Website.pdfAdditional Benefits for Employee Website.pdf
Additional Benefits for Employee Website.pdf
 
Thesis Statement for students diagnonsed withADHD.ppt
Thesis Statement for students diagnonsed withADHD.pptThesis Statement for students diagnonsed withADHD.ppt
Thesis Statement for students diagnonsed withADHD.ppt
 
1.4 modern child centered education - mahatma gandhi-2.pptx
1.4 modern child centered education - mahatma gandhi-2.pptx1.4 modern child centered education - mahatma gandhi-2.pptx
1.4 modern child centered education - mahatma gandhi-2.pptx
 
Supporting (UKRI) OA monographs at Salford.pptx
Supporting (UKRI) OA monographs at Salford.pptxSupporting (UKRI) OA monographs at Salford.pptx
Supporting (UKRI) OA monographs at Salford.pptx
 
How to Make a Field invisible in Odoo 17
How to Make a Field invisible in Odoo 17How to Make a Field invisible in Odoo 17
How to Make a Field invisible in Odoo 17
 
The geography of Taylor Swift - some ideas
The geography of Taylor Swift - some ideasThe geography of Taylor Swift - some ideas
The geography of Taylor Swift - some ideas
 
Overview on Edible Vaccine: Pros & Cons with Mechanism
Overview on Edible Vaccine: Pros & Cons with MechanismOverview on Edible Vaccine: Pros & Cons with Mechanism
Overview on Edible Vaccine: Pros & Cons with Mechanism
 
CLASS 11 CBSE B.St Project AIDS TO TRADE - INSURANCE
CLASS 11 CBSE B.St Project AIDS TO TRADE - INSURANCECLASS 11 CBSE B.St Project AIDS TO TRADE - INSURANCE
CLASS 11 CBSE B.St Project AIDS TO TRADE - INSURANCE
 
The French Revolution Class 9 Study Material pdf free download
The French Revolution Class 9 Study Material pdf free downloadThe French Revolution Class 9 Study Material pdf free download
The French Revolution Class 9 Study Material pdf free download
 
Home assignment II on Spectroscopy 2024 Answers.pdf
Home assignment II on Spectroscopy 2024 Answers.pdfHome assignment II on Spectroscopy 2024 Answers.pdf
Home assignment II on Spectroscopy 2024 Answers.pdf
 

Basic stat

  • 1. 1. INTRODUCTION TO STATISTICS  The impact of statistics profoundly affects society today. Every day, we have confronted with some form of statistical information through news papers, magazines, and other forms of communication. Besides, statistical tables, survey results, and the language of probability are used with increasing frequency by the media. Such statistical information has become highly influential in our lives. Statistics can be considered as numerical statements of facts which are highly convenient and meaningful forms of communication. The subjects of statistics, as it seems, is not a new discipline but it is as old as the human society itself. The sphere of its utility, however, was very much restricted. The word statistics is derived from the Latin word status which means a political state or government. It was originally applied in connection with kings and monarchs collecting data on their citizenry which pertained to state wealth, taxes collected population and so on. Thus, the scope of statistics in the ancient times was primarily limited to the collection of demographic and property and wealth data of a country by governments for framing military and fiscal policies. Now days, statistics is used almost in every field of study, such as natural science, social science, engineering, medicine, agriculture, etc. The improvements in computer technology make it easier than ever to use statistical methods and to manipulate massive amounts of data. Definition and Classification of Statistics Statistics as a subject (field of study): Statistics is defined as the science of collecting, organizing, presenting, analyzing and interpreting numerical data to make decision on the bases of such analysis.(Singular sense) Statistics as a numerical data: Statistics is defined as aggregates of numerical expressed facts (figures) collected in a systematic manner for a predetermined purpose. (Plural sense) In this course, we shall be mainly concerned with statistics as a subject, that is, as a field of study. Classification of Statistics: Statistics is broadly divided into two categories based on how the collected data are used. i. Descriptive Statistics: is concerned with summary calculations, graphs, charts and tables. It deals with describing data without attempting to infer anything that goes beyond the given set of data. It consists of collection, organization, summarization and presentation of data in the convenient and informative way. Examples: • The amount of medication in blood pressure pills. • The starting salaries for Mathematics and Statistics students in different organizations. 1   
  • 2. ii. Inferential Statistics: Descriptive Statistics describe the data set that’s being analyzed, but doesn’t allow us to draw any conclusions or make any inferences about the data, other than visual “It looks like …..” type statements. It deals with making inferences and/or conclusions about a population based on data obtained from a limited sample of observations. It consists of performing hypothesis testing, determining relationships among variables and making predictions. For example, the average income of all families (the population) in Ethiopia can be estimated from figures obtained from a few hundred (the sample) families. Examples: a) From past figures, it has been predicted that 90 0 0 of registered voters will vote in the November election. b) The average age of a student in Hawassa University is 19.1 years. c) To determine the most effective dose of a new medication (on the basis of tests performed with volunteer patients from selected hospitals) d) To compare the effectiveness of two reducing diets (based on the weights losses of persons who were taking the diets) e) There is a relationship between smoking tobacco and an increased risk of developing cancer. Stages of Statistical Investigation The area of statistics incorporates the following five stages. These are collection, organization, presentation, analysis and interpretation of data. Proper collection of data: Data can be collected in a variety of ways; one of the most common methods is through the use of survey. Survey can also be done in different methods, three of the most common methods are: Telephone survey, Mailed questionnaire, Personal interview. Organization of data: If an investigator has collected data through a survey, it is necessary to edit these data in order to correct any apparent inconsistencies, ambiguities, and recording errors. Presentation of data: the organized data can now be presented in the form of tables or diagrams or graphs. This presentation in an orderly manner facilitates the understanding as well as analysis of data. Analysis of data: the basic purpose of data analysis is to make it useful for certain conclusions. This analysis may simply be a critical observation of data to draw some meaningful conclusions about it or it may involve highly complex and sophisticated mathematical techniques. Interpretation of data: Interpretation means drawing conclusions from the data which form the basis of decision making. Correct interpretation requires a high degree of skill and experience and is necessary in order to draw valid conclusions. 2   
  • 3. Definition of some basic terms Population: It is the totality of things under considerations. The population represents the target of an investigation, and the objective of the investigation is to draw conclusions about the population hence we sometimes call it target population. Examples: • All university students of Ethiopia • Population of trees under specified climatic conditions • Population of animals fed a certain type of diet • Population of farms having a certain type of natural fertility • Population of households, etc The population could be finite or infinite (an imaginary collection of units). There are two ways of investigation: Census and sample survey. Census: a complete enumeration of the population. But in most real problems it cannot be realized, hence we take sample. Sample: A group of subjects selected from a population. A sample from a population is the set of measurements that are actually collected in the course of an investigation. For example, if we want to study the income pattern of lecturers at Hawassa University and there are 3000 lecturers, then we may take a random sample of only 250 lecturers out of this entire population of 3000 for the purpose of study. Then this number of 250 lecturers constitutes a sample. In practice, we rarely conduct census, instead we conduct sample survey Parameter: is a descriptive measure of a population, or summary value calculated from a population. Examples: Average, Range, proportion, variance, Statistic: is a descriptive measure of a sample, or summary value calculated from a sample. From the previous example, the summary measure that describes a characteristic such as average income of this sample is known as a statistic. Sampling: The process or method of sample selection from the population. Sample size: The number of elements or observation to be included in the sample. Variable: It is an item of interest that can take on many different numerical values. Applications, Uses and Limitation of Statistics Applications of Statistics Statistics can be applied in any field of study which seeks quantitative evidence. For instance, engineering, economics, natural science, etc. a) Engineering: Statistics have wide application in engineering. • To compare the breaking strength of two types of materials • To determine the probability of reliability of a product. • To control the quality of products in a given production process. • To compare the improvement of yield due to certain additives (fertilizer, herbicides, (wee decides), e t c b) Economics: Statistics are widely used in economics study and research. • To measure and forecast Gross National Product (GNP) 3   
  • 4. • Statistical analyses of population growth, unemployment figures, rural or urban population shifts and so on influence much of the economic policy making • Financial statistics are necessary in the fields of money and banking including consumer savings and credit availability. c) Statistics and research: there is hardly any advanced research going on without the use of statistics in one form or another. Statistics are used extensively in medical, pharmaceutical and agricultural research. Uses of Statistics Today the field of statistics is recognized as a highly useful tool to making decision process by managers of modern business, industry, frequently changing technology. It has a lot of functions in everyday activities. The following are some uses of statistics: • Statistics condenses and summarizes complex data: the original set of data (raw data) is normally voluminous and disorganized unless it is summarized and expressed in few numerical values. • Statistics facilitates comparison of data: measures obtained from different set of data can be compared to draw conclusion about those sets. Statistical values such as averages, percentages, ratios, etc, are the tools that can be used for the purpose of comparing sets of data. • Statistics helps in predicting future trends: statistics is extremely useful for analyzing the past and present data and predicting some future trends. • Statistics influences the policies of government: statistical study results in the areas of taxation, on unemployment rate, on the performance of every sort of military equipment, etc, may convince a government to review its policies and plans with the view to meet national needs and aspirations. • Statistical methods are very helpful in formulating and testing hypothesis and to develop new theories. Limitations of Statistics The field of statistics, though widely used in all areas of human knowledge and widely applied in a variety of disciplines such as business, economics and research, has its own limitations. Some of these limitations are: a) It does not deal with individual values: as discussed earlier, statistics deals with aggregate of values. For example, the population size of a country for some given year does not help us for comparative studies, unless it can be compared with some other countries or with a set standard. b) It does not deal with qualitative characteristics directly: statistics is not applicable to qualitative characteristics such as beauty, honesty, poverty, standard of living and so on since these cannot be expressed in quantitative terms. These characteristics, however, can be statistically dealt with if some quantitative values can be assigned to these with logical criterion. For 4   
  • 5. c) Statistical conclusions are not universally true: since statistics is not an exact science, as is the case with natural sciences, the statistical conclusions are true only under certain assumptions. Also, the field deals extensively with the laws of probability which at best are educated guesses. For example, if we toss a coin 10 times where the chances of a head or a tail are 1:1, we cannot say with certainty that there will be 5 heads and 5 tails. Thus the statistical laws are only approximations. d) It is sensitive for misuse: The famous statement that figures don’t lie but the liars can figure, is a testimony to the misuse of statistics. Thus inaccurate or incomplete figures can be manipulated to get desirable references. The number of car accidents committed in a city in a particular year by women drivers is 10 while that committed by men drivers is 40. Hence women drivers are safe drivers. TYPES OF VARIABLES AND MEASUREMENT SCALES Types of Variables A variable is a characteristic of an object that can have different possible values. Age, height, IQ and so on are all variables since their values can change when applied to different people. For example, Mr. X is a variable since X can represent anybody. A variable may be qualitative or quantitative in nature. These two types of variables are: a) Quantitative variables: are variables that can be quantified or can have numerical values. Examples: height, area, income, temperature e t c. Consider the following questions; How many rooms are there in your house? Or How many children are there in the family? Would be in numerical values. b) Qualitative variables: are variables that cannot be quantified directly. Examples: color, beauty, sex, location, political affiliation, and so on. Consider also the following questions; “are you currently unemployed?” Would fit in the category of either yes or no. Qualitative variables are also called categorical variables. And hence we have two types of data; quantitative & qualitative data. Quantitative variables can be further classified as Discrete variables, and Continuous variables a) Discrete variables are variables whose values are counts. For examples: • number of students attending a conference • number of households (family size) • Number of pages of a book • number of eggs in the refrigerator, etc b) Continuous variables are variables that can have any value within an interval. 5   
  • 6. Examples: • height of models in a beauty context • weight of people joining a diet program • Lengths of steel bars in a given production terms, e t c. Scales of Measurement Proper knowledge about the nature and type of data to be dealt with is essential in order to specify and apply the proper statistical method for their analysis and inferences. Measurement is the assignment of numbers to objects or events in a systematic  fashion.  It is a functional mapping from the set of objects {Oi} to the set of real numbers {M(Oi)} Measurement scale refers to the property of value assigned to the data based on the properties of order, distance and fixed zero. Four levels of measurement scales are commonly distinguished: nominal, ordinal, interval, and ratio and each possessed different properties of measurement systems  Nominal scale: - “Nominal “is a Latin word for “name” which measures the presence or absence of a characteristic of the data. The values of a nominal attribute are just different names, i.e., nominal attributes provide only enough information to distinguish one object from another. Qualities with no ranking or ordering; no numerical or quantitative value. Data consists of names, labels and categories. This is a scale for grouping individuals into different categories. Examples: Car colors for a certain model are: red, silver, blue and black. • In this scale, one is different from the other • +, -, *, /, Impossible, comparison is impossible Ordinal scale: - measures the presence or absence of a characteristic of the data using an implied ranking or order. “Ordinal” is a Latin word, meaning “order”. • Can be arranged in some order, but the differences between the data values are meaningless. • Data consisting of an ordering of ranking of measurements are said to be on an ordinal scale of measurements. That is, the values of an ordinal scale provide enough information to order objects. (<, >) • One is different from and grater /better/ less than the other • +, -, *, / Are impossible, comparison is possible. Consider the following examples about ordinal scale: a. Man A weighs more than man B b. Ethiopian athletes got 1st and 2nd ranks in the 10,000m women’s final in Beijing. c. Of 17 fishing reels rated: 6 were rated good quality, 4 were rated better quality, and 7 were rated best quality. d. Out of a high school class of 319, Walter ranked 4th, June ranked 12th, and Jim ranked 20th. 6   
  • 7. Interval Level: Data values can be ranked and the differences between data values are meaningful. However, there is no intrinsic zero, or starting point, and the ratio of data values are meaningless. Note: Celsius & Fahrenheit temperature readings have no meaningful zero and ratios are meaningless. In this measurement scale:- • One is different, better/greater by a certain amount of difference than another • Possible to add and subtract. For example; 37O c –35o c = 2o c, 45o c – 43 o c= 2o c • Multiplication and division are not possible. For example; 40o c = 2(20o c) But this does not imply that an object which is 40 o c is twice as hot as an object which is 20 o c Interval scale data convey better information than nominal and ordinal scale. Most common examples are: IQ, temperature, Calendar dates, etc Consider the following examples: a. The years in which democrats won presidential elections. b. Body temperature in degrees Celsius (or Fahrenheit) c. Building A was built in 1284, Building B in 1492 and Building C in 5 B.C. Ratio scale: Similar to interval, except there is a true zero, or starting point, and the ratios of data values have meaning. Best type of data: can add, subtract, multiply and divide. For ratio variables, both differences and ratios are meaningful. One is different, larger /taller/ better/ less by a certain amount of difference and so much times than the other. This measurement scale provides better information than interval scale of measurement Examples: temperature in Kelvin, monetary quantities, counts, age, mass, length, electric current. Consider the following some more examples: a. Core temperatures of stars measures in degrees Kelvin b. Time elapsed between the deposit of a check and the clearance of that check. c. Length of the Nile River. 2. Methods of Data Collection and Presentation  Methods of Data Collection Data: are the values (measurements or observations) that the variables can assume. Variables that are determined by chance are called random variables. Any aggregate of numbers cannot be called statistical data. We say an aggregate of numbers is statistical data when they are • Comparable • Meaningful and • Collected for a well defined objective Raw data: are collected data, which have not been organized numerically. Examples: 25, 10, 32, 18, 6, 93, 4. 7   
  • 8. The required data can be obtained from either a primary source or a secondary source. Primary source: Is a source of data that supplies first hand information for the use of the immediate purpose. Primary Data: data you collect to answer your question. Data measured or collected by the investigator or the user directly from the source. Or data originally collected for the immediate purpose. Two activities involved: planning and measuring. a) Planning: • Identify source and elements of the data. • Decide whether to consider sample or census. • If sampling is preferred, decide on sample size, selection method,… etc • Decide measurement procedure. • Set up the necessary organizational structure. b) Measuring: there are different options. • Focus Group • Telephone Interview • Mail Questionnaires • Door-to-Door Survey • Mall Intercept • New Product Registration • Personal Interview and • Experiments are some of the sources for collecting the primary data. Secondary source: are individuals or agencies, which supply data originally collected for other purposes by them or others. Usually they are published or unpublished materials, records, reports, e t c. Secondary data: data collected from a secondary source by other people for other purposes. Data gathered or compiled from published and unpublished sources or files. When our source is secondary data check that: • The type and objective of the situations. • The purpose for which the data are collected and compatible with the present problem. • The nature and classification of data is appropriate to our problem. • There are no biases and misreporting in the published data. Note: Data which are primary for one may be secondary for the other. Methods of Data Presentation Having collected and edited the data, the next important step is to organize it. That is to present it in a readily comprehensible condensed form that aids in order to draw inferences from it. It is also necessary that the like be separated from the unlike ones. The presentation of data is broadly classified in to the following two categories: • Tabular presentation • Diagrammatic and Graphic presentation. The process of arranging data in to classes or categories according to similarities technically is called classification. It eliminates inconsistency and also brings out the 8   
  • 9. points of similarity and/or dissimilarity of collected items/data. It is necessary because it would not be possible to draw inferences and conclusions if we have a large set of collected [raw] data. Frequency Distributions Frequency: - is the number of times a certain value or set of values occurs in a specific group. A frequency distribution is a table that presents data according to some criteria with the corresponding number of items falling in each class (i.e. with the corresponding frequencies.). We see at a glance the shape of the distribution, the range of variation, and any clustering of the values. By presenting a frequency distribution in relative form, i.e., as percentages, we convert to the familiar base of 100 and make it easy to compare the distribution of cases between different variables and/or different samples, each of which may involve different total numbers of cases. Generally, there are three basic types of frequency distributions: Categorical, Ungrouped and Grouped frequency distributions. 1. Categorical frequency distribution – the data are usually qualitative – the scales of measurements for the data are usually nominal or ordinal For instance data on blood types of people, political affiliation, economic status (low, medium and high), religious affiliation are presented by categorical frequency distributions. Example: Thirty students, last year, took Stat 100 course and their grades were as follows. Construct an appropriate frequency distribution for these data. Table 2.1: Grades of students B B C B A C D C C C B B B A B C D C A F B F C A B C C A C D Solution: There are five kinds of grades: A, B, C, D and F which may be used as the classes for constructing the distribution. The procedure for constructing a frequency distribution for categorical data is given below. STEP 1. Construct a table as shown below STEP 2. Tally the data and place the results STEP 3. Count the Tallies and put the results 9   
  • 10. STEP 4. Calculate the percentages (%) of frequencies in each class ( ) 100% ×= n f Where = frequency of the class = total number of observationsf n Class ( I ) Tally ( II ) Frequency ( III ) Percent* ( IV ) A ///// 5 16.7 B ///// //// 9 30.0 C ///// ///// / 11 36.7 D /// 3 10.0 F // 2 6.7 2. Ungrouped frequency distribution Ungrouped frequency distribution is a table of all potential raw scored values that could possibly occur in the data along with their corresponding frequencies. It is often constructed for small set of data or a discrete variable. Constructing an ungrouped frequency distribution To construct an ungrouped frequency distribution, first find the smallest and the largest raw scores in the collected data. Then make a columnar table of all potential raw scored values arranged in order of magnitude with the number of times a particular value is repeated, i.e., the frequency of that value. To facilitate counting method, tallies can be used. Example: The following data are the ages in years of 20 women who attend health education last year: 30, 41, 39, 41, 32, 29, 35, 31, 30, 36, 33, 36, 32, 42, 30, 35, 37, 32, 30, and 41. Construct a frequency distribution for these data. Solution: STEP 1. Find the range: nobservatioMinimumnobservatioMaximumRange −= STEP 2. Construct a table, tally the data and complete the frequency column. The frequency distribution becomes as follows. Age 29 30 31 32 33 35 36 37 39 41 42 Tally / //// / /// / // // / / /// / Frequency 1 4 1 3 1 2 2 1 1 3 1 3. Grouped frequency distribution STEP 1. Determine the unit of measurement, U STEP 2. Find the maximum(Max) and the minimum(Min) observation, and then compute their range, R MinMaxRange −= STEP 3. Fix the number of classes’ desired (k). there are two ways to fix k: • Fix k arbitrarily between 6 and 20, or • Use Sturge’s Formula: Nk 10log332.31+= where N is the total frequency. And round this value of k up to get an integer number. 10   
  • 11. STEP 4. Find the class widths (W) by dividing the range by the number of classes and round the number up to get an integer value. K RW = STEP 5. Pick a suitable starting point less than or equal to the minimum value. This starting point is the lower limit of the first class. Continue to add the class width to this lower limit to get the rest of the lower limits. STEP 6. Find the upper class limits. To find the upper class limit of the first class, subtract one unit of measurement from the lower limit of the second class. Then continue to add the class width to this upper limit to get the rest of the upper limits. STEP 7. Compute the class boundaries as: ULCLLCB 2 1 −= and UUCLUCB 2 1 += Where LCL = lower class limit, UCL= upper class limit, LCB= lower class boundary and UCB= upper class boundary. The class boundaries are also half way between the upper limit of one class and the lower limit of the next class. STEP 8. Tally the data. STEP 9. Find the frequencies. STEP 10. (If necessary) Find the cumulative frequencies (more than and less than types). Example: The number of hours 40 employees spends on their job for the last 7 days Table: Number of hours employees spend 62 50 35 36 31 43 43 43 41 31 65 30 41 58 49 41 37 62 27 47 65 50 45 48 27 53 40 29 63 34 44 32 58 61 38 41 26 50 47 37 Construct a suitable frequency distribution for these data using 8 classes Solution: 1. Unit of measurement; U= 1year 2. Max = 65, Min = 26 so that R = 65-26 = 39 3. It is already determined to construct a frequency distribution having 8 classes. 4. Class width 5875.4 5 39 ≈==W 5. Starting point = 26 = lower limit of the first class. And hence the lower class limits become: 26 31 36 41 46 51 56 61 6. Upper limit of the first class = 31-1 = 30. And hence the upper class limits become 30 35 40 45 50 55 60 65 The lower and the upper class limits (Steps 5 and 6) can be written as follows. Class limits 26-30 31-35 36-40 41-45 46-50 51-55 56-60 61-65 7. By subtracting 0.5 units of measurement from the lower class limits and by adding 0.5 units of measurement to the upper class limits, we can get lower and upper class boundaries as follows. Class Boundaries 25.5- 30.5 30.5- 35.5 35.5- 40.5 40.5- 45.5 45.5- 50.5 50.5- 55.5 55.5- 60.5 60.5- 65.5 STEPS 8, 9 and 10 are displayed in the following table. 11   
  • 12. Class limits Class boundaries Tally frequ ency Cumulative frequency (less than type) Cumulative frequency (more than type) 26 – 30 25.5 – 30.5 ///// 5 5 40 31 – 35 30.5 – 35.5 ///// 5 10 35 36 – 40 35.5– 40.5 ///// 5 15 30 41 – 45 40.5– 45.5 ///// //// 9 24 25 46 – 50 45.5– 50.5 ///// // 7 31 16 51 – 55 50.5– 55.5 / 1 32 9 56 – 60 55.5– 60.5 // 2 34 8 61 – 65 60.5– 65.5 ///// / 6 40 6 Diagrammatic presentation of data: Bar charts, Pie-chart, Cartograms The most convenient and popular way of describing data is using graphical presentation. It is easier to understand and interpret data when they are presented graphically than using words or a frequency table. A graph can present data in a simple and clear way. Also it can illustrate the important aspects of the data. This leads to better analysis and presentation of the data. What it is? Graphs are pictorial representations of the relationships between two (or more) variables and are an important part of descriptive statistics. Different types of graphs can be used for illustration purposes depending on the type of variable (nominal, ordinal, or interval) and the issues of interest. When to Use It? Graphs can be used any time one wants to visually summarize the relationships between variables, especially if the data set is large or unmanageable. They are routinely used in reports to underscore a particular statement about a data set and to enhance readability. Graphs can appeal to visual memory in ways that mere tallies, tables, or frequency distributions cannot. However, if not used carefully, graphs can misrepresent relationships between variables or encourage inaccurate conclusions. Importance: • They have greater attraction. • They facilitate comparison. • They are easily understandable. The three most commonly used diagrammatic presentation for discrete as well as qualitative data are: • Pie charts, • Pictogram, • Bar charts Pie chart A pie chart is a circle that is divided in to sections or wedges according to the percentage of frequencies in each category of the distribution. The angle of the sector is obtained using: 360* o quantitywholethe parttheofValue SectorofAngle = 12   
  • 13. Example The following data are the blood types of 50 volunteers at a blood plasma donation clinic: O A O AB A A O O B A O A AB B O O O A B A A O A A O B A O AB A O O A B A A A O B O O A O A B O AB A O B a) Organize this data using a categorical frequency distribution b) Present the data using both a pie and a bar chart. Solution a) The classes of the frequency distribution are A, B, O, AB. Count the number of donors for each of the blood types. Table: the number of donors by blood types. Blood type Frequency Percent A 19 38 B 8 16 O 19 38 AB 4 8 Total 50 100 b) Pie chart Find the percentage of donors for each blood type. In order to find the angles of the sector for each blood type, multiply the corresponding percentage by 3600 and divide by 100. Figure: Pie-chart of the data on blood types of donors. AB O B A Blood type Pictogram In these diagrams, we represent data by means of some picture symbols. We decide abut a suitable picture to represent a definite number of units in which the variable is measured Example: The following table shows the orange production in a plantation from production year 1990-1993. Represent the data by a pictogram. Solution: Table: Orange productions from 1990 to 1993 Production year 1990 1991 1992 1993 Amount (in kg) 3000 3850 3500 5000 13   
  • 14. Figure: Pictogram of the data on Orange productions from 1990 to 1993. Bar Charts Bar graphs are commonly used to show the number or proportion of nominal or ordinal data which possess a particular attribute. They depict the frequency of each category of data points as a bar rising vertically from the horizontal axis. Bar graphs most often represent the number of observations in a given category, such as the number of people in a sample falling into a given income or ethnic group. They can be used to show the proportion of such data points. Bar graphs are especially good for showing how nominal data change over time. They are useful for comparing aggregate over time space. Bars can be drawn either vertically or horizontally. There are different types of bar charts. The most common being: Simple bar chart, Component or sub divided bar chart, and Multiple bar charts. Simple Bar Chart: Are used to display data on one variable and are thick lines (narrow rectangles) having the same breadth. Example: Suppose that the following were the gross revenues (in $100,000.00) for company XYZ for the years 1989, 1990 and 1991. Draw the bar graph for this data. Table: Gross revenue of companies Year Revenue 1989 110 1990 95 1991 65 Solution: The bar diagram for this data can be constructed as follows with the revenues represented on the vertical axis and the years represented on the horizontal axis. 199119901989 110 100 90 80 70 60 Year SumofRevenue 14   
  • 15. Component Bar chart When there is a desire to show how a total (or aggregate) is divided in to its component parts, we use component bar chart. The bars represent total value of a variable with each total broken in to its component parts and different colors or designs are used for identifications Example: Construct a sub-divided bar chart for the four types of products in relation to the opinion of consumers purchasing the given products as given below: Table: Opinion of consumers purchasing the given products Products Definitely Probably Unsure No Product 1 50% 40% 10% 2% Product 2 60% 30% 12% 15% Product 3 70% 45% 8% 8% Product 4 60% 35% 5% 20% Component Bar Chart 0% 50% 100% 1 2 3 4 Products Opinionof Consumers NO Unsure Probably Definitely Multiple Bar charts These are used to display data on more than one variable. They are used for comparing different variables at the same time. Example: Construct a sub-divided bar chart for the 3 types of expenditures in dollars for a family of four for the years 1, 2, 3 and 4 as given below: Table: Expenditures in dollars for families Year Food Education Other Total 1 3000 2000 3000 8000 2 3500 3000 4000 10500 3 4000 3500 5000 12500 5000 5000 6000 16000 Bar Charts 0 2000 4000 6000 8000 1 2 3 4 Year Typesof expenditures Food Education Other Total Figure: Multiple bar chart for Expenditures in dollars for families 15   
  • 16. Graphical presentation of data: Histogram, and Frequency Polygon The histogram, frequency polygon and cumulative frequency graph or ogives are most commonly applied graphical representation for continuous data. Procedures for constructing statistical graphs: • Draw and label the X and Y axes. • Choose a suitable scale for the (cumulative) frequencies and label it on the Y axes. • Represent the class boundaries for the histogram or ogive or the mid points for the frequency polygon on the X axes. • Plot the points. • Draw the bars or lines to connect the points. i. Histogram Histograms or column bar charts are common ways of presenting frequency in a number of categories. Commonly used graphical presentation methods also include the frequency polygon and ogive. Histograms portray an unequal width frequency distribution table for further statistical use. A frequency distribution table is a tabulation of the n measurements into mutually exclusive k classes showing the number of observations in each. The bars appear in a histogram where the classes are marked on the x axis and the class frequencies on the y axis. The histogram is constructed by creating x-axis units of equal size and these should correspond to the frequency table. Histogram usually shows the number of observations in a specific range. Example: Construct a histogram for the frequency distribution of the time spent by the automobile workers. Table: Time in minutes spent by automobile workers Time (in minute) Class mark Number of workers 15.5- 21.5 18.5 3 21.5-27.5 24.5 6 27.5-33.5 30.5 8 33.5-39.5 36.5 4 39.5-45.5 42.5 3 45.5-51.5 48.5 1 0 2 4 6 8 10 18.5 24.5 30.5 36.5 42.5 48.5 Frequency Time Figure: Histogram of the data on the number of minutes spent by the automobile workers. 16   
  • 17. Table: Data given to present a Histogram Class Boundaries Class Mark Frequency Cumulative Frequency (less than type) Cumulative Frequency (more than type) 5.5 – 11.5 8.5 2 2 20 11.5 – 17.5 14.5 2 4 18 17.5 – 23.5 20.5 7 11 16 23.5 – 29.5 26.5 4 15 9 29.5 – 35.5 32.5 3 18 5 35.5 – 41.5 38.5 2 20 2 ii. Frequency Polygon A frequency polygon is a line graph drawn by taking the frequencies of the classes along the vertical axis and their respective class marks along the horizontal axis. Then join the cross points by a free hand curve. Example: Construct a frequency polygon for the frequency distribution of the time spent by the automobile workers that we have seen in example 2.9. Figure: Number of minutes spent by the automobile workers. iii. Cumulative Frequency Polygon (Ogive) It is a graph obtained by plotting the cumulative frequencies of a distribution against the boundaries used to form the cumulative frequencies. Example: Construct an ogive for the time spent by the automobile workers. 17   
  • 18. 3. Measures of Central Tendency Introduction Measures of Central Tendency give us information about the location of the center of the distribution of data values. A single value that describes the characteristics of the entire mass of data is called measures of central tendency. We will discuss briefly the three measures of central tendency: mean, median and mode in this unit. Suppose we have a random sample of n values of some measurement X. The values are X1 , X2 , . . ,Xn. We want to summarize the information contained in this sample as regards an "average" level. An average is a numerical value that indicates the middle point or central region of the raw data. Mathematically summarize data in order to make appropriate comparisons. Objectives of measures of central tendency Objectives of measuring central tendency are: • To get a single value that represent(describe) characteristics of the entire data • To summarizing/reducing the volume of the data • To facilitating comparison within one group or between groups of data • To enable further statistical analysis The Summation Notation (∑): Let a data set consists of a number of observations, represents by where n denotes the number of observations in the data and is the ith observation. Then, is the sum of all the observation∑=i ix 1 N ∑ xxx n.,..,, 21 ix For instance a data set consisting of six measurements 21, 13, 54, 46, 32 and 37 is represented by and where = 21, = 13, = 54, = 46, = 32 and = 37. 54321 ,,,, xxxxx 6x 1x 2x 3x 4x 5x 6x Their sum becomes 21+13+59+46+32+37=208.∑= = 6 1i ix Similarly = 22 2 2 1 ... nxxx +++ ∑= n i ix1 2 Some Properties of the Summation Notation 1. = where c is a constant number.∑= n i c 1 cn. 2. where is a constant number∑ ∑= = = n i n i ii xbxb 1 1 . b 3. where a and b are constant numbers∑= = +=+ n i n i ii xbanbxa 1 1 .)( 4. ∑∑∑ === ±=± n i i n i i n i ii yxyx 111 )( 5. ∑∑∑ === ≠ n i i n i i n i ii yxyx 111 18   
  • 19. Important Characteristics of a Good Average: A typical average should posses the  following:   • It should be rigidly defined. • It should be based on all observation under investigation. • It should be as little as affected by extreme observations. • It should be capable of further algebraic treatment. • It should be as little as affected by fluctuations of sampling. • It should be ease to calculate and simple to understand. i. Mean The mean of a sample is determined by summing up all the data values of the sample and dividing this sum by the total number of data values. There are four types of means which are suitable for a particular type of data. These are 1. Arithmetic Mean 2. Geometric Mean 3. Harmonic Mean Arithmetic Mean Arithmetic mean is defined as the sum of the measurements of the items divided by the total number of items. It is usually denoted by x. Suppose are n observed values in a sample of size n from a population of size N, n<N then the arithmetic mean of the sample, denoted by xx n ..,, 21x x is given by n x n x....xx = in21 ∑= +++ X If we take an entire population Mean is denoted by µ and is given by: NN = N21 =μ xx....xx i∑+++ Example: Consider the samples given for the two routes as follows: For route A, the sample values are: 38 41 34 46 36 30 37 36 32 40 For route B the sample values are: 28 32 36 30 31 37 36 80 32 34 31 Find the arithmetic mean for the two routes. Solution: For route A, the sample values were: 38 41 34 46 36 30 37 36 32 40. 0.37 10 370 10 40+32+36+37+30+36+46+34+14+38 =x == minutes For route B the sample values were: 28 32 36 30 31 37 36 80 32 34 31. 0.37 11 407 11 3134+32+80+36+37+31+30+36+32+28 =x == + minutes 19   
  • 20. When the numbers occur with frequencies , respectively, then mean can be expressed in a more compact form as , ,   , … ,   , ,   , … ,     ∑ ∑ Example Calculate the arithmetic mean of the sample of numbers of people living in twelve houses in a street: 3 2 4 4 3 1 2 5 3 3 8 1. 25.3 12 39 12 183+3+5+2+1+3+4+4+2+3 =x == + + The information in the sentence above can be written in a table, as follows. Value, x 1 2 3 4 5 8 Frequency, f 2 2 4 2 1 1   The formula for the arithmetic mean for data of this type is ∑ ∑ In this case we have: .25.3 12 39 12 8581242 112422 81514241 == 3222 +++++ = +++++ +×+ =x ×+×+× × + × The mean numbers of people living in twelve houses in a street is 3.25. Arithmetic Mean for Grouped Frequency Distribution If data are given in the form of continuous frequency distribution, the sample mean is: fff mfmfmf f mf k kk k i i i ii x +++ +++ == ∑ ∑ = = ... ... 21 2211 1 1 i nf i i =∑=1 th i k Where m is the class mark of the class; i = 1, 2, …, k; th fi is the frequency of the i class and k = the number of classes k Note that = the total number of observations. Example: The following table gives the height (inches) of 100 students in a college. Class Interval (CI) Frequency (f) 60 - 62 5 62 – 64 18 64 – 66 42 66 – 68 20 68 – 70 8 70 – 72 7 Total 100 Calculate the mean? 20   
  • 21. Solution: ∑ ∑ = = = k i i k i ii f mf x 1 1 100 1 ==∑= nf k i i , ∑ ∑ = = = k i i k i ii f mf x 1 1 = 100 6558 = 65.58 The mean height of students is 65.58 Properties of the Arithmetic Mean The sum of the deviations of the items from their arithmetic mean is zero. This means, the algebraic sum of the deviations of a set of numbers from their mean nxxx .,..,, 21 x is zero. That is The sum of the squares of the deviations of a set of observations from any number, say A, is minimum when A= . That is, ∑   ∑ 0)( 1 =−∑= n i i xx When a set of observations is divided into k groups and 1x the mean of 1n servations of group 1, is ob 2x the mean of n bservations of group2, …,is 2 o kx is the mean of observations of group k , then the combined mean ,denoted bykn x , of all observations taken together is given by If the mean of isnxxx .,..,, 21 x , then a) the mean of kxkxkx n ±±± .,..,, 21 will be kx ± b) The mean of will benkxkxkx .,..,, 21 xk . Example Last year there were three sections taking Stat 200 course. At the end of the semester, the three sections got average marks of 80, 83 and 76. There were 28, 32 and 35 students in each section respectively. Find the mean mark for the entire students. Solution: == ++ ++ = ++ ++ = 95 7556 353228 )76(35)83(32)80(28 321 332211 nnn xnxnxn x 79.54 Weighted Arithmetic Mean In finding arithmetic mean, all items were assumed to be of equally importance (each value in the data set has equal weight). When the observations have different weight, we use weighted average. Weights are assigned to each item in proportion to its relative importance. 21   
  • 22. If represent values of the items and are the corresponding weights, then the weighted mean, kxxx .,..,, 21 kwww ,...,, 21 )( wx is given by 1 1 2 2 1 1 2 1 ... ... k i i k k i w k k i i w x w x w x w x X w w w w = = + + + = = + + + ∑ ∑ Example: A student’s final mark in Mathematics, Physics, Chemistry and Biology are respectively 82, 80, 90 and 70.If the respective credits received for these courses are 3, 5, 3 and 1, determine the approximate average mark the student has got for one course. Solution We use a weighted arithmetic mean, weight associated with each course being taken as the number of credits received for the corresponding course. Therefore 17.82 1353 )701()903()805()823( = +++ ×+×+×+× == ∑ ∑ i ii w w xw x Average mark of the student for one course is approximately 82. Geometric Mean The geometric mean like arithmetic mean is calculated average. It used when observed values are measured as ratios, percentages, proportions, indices or growth rates. The geometric mean, G.M. of a series of numbers is defined asxn xx .,..,, 21 n nxxxGM ..... 21 = , Properties of geometric mean a. Its calculations are not as such easy. b. It involves all observations during computation c. It may not be defined even it a single observation is negative. d. If the value of one observation is zero its values becomes zero. Remark: 1) When the observed values have the corresponding frequenciesxn xx .,..,, 21 f n ff .,..,, 21 respectively, the geometric mean is obtained by n kxxx fffGM k..... 21 21 = 2) The above formula can also be used whenever the frequency distribution are grouped (continuous), class marks of the class intervals are considered as im n kmmm fffGM k..... 21 21 = Where ∑= = n i i fn 1 Example: Compute the geometric mean of the following values: 2, 8, 6, 4, 10, 6, 8, 4 Solution: 22   
  • 23. n kxxx fffGM k..... 21 21 = = 8 1221 108642 ...2. 8 106436162 xxxxGM = 8 737280= = 5.41 The geometric mean for the given data will be 5.41 Harmonic Mean It is a suitable measure of central tendency when the data pertains to speed, rate and time. The harmonic mean is defined as the reciprocal of the arithmetic mean of the reciprocal of the individual observations. If are n observations, then harmonic mean can be represented by the following formula: xn xx .,..,, 21 xxx n n i i nn HM 1 .... 11 1 1 ++ == ∑ = If the data arranged in the form of frequency distribution xfxf ff xf f kk k n i i k i i i HM ++ ++ == ∑ ∑ = = ........ 1 ..... 1 11 1 1 1 Properties of harmonic mean i. It is based on all observation in a distribution. ii. Used when a situations where small weight is give for larger observation and larger weight for smaller observation iii. Difficult to calculate and understand iv. Appropriate measure of central tendency in situations where data is in ratio, speed or rate Example: A motorist travels 480km in 3 days. She travels for 10 hours at rate of 48km/hr on 1st day, for 12 hours at rate of 40km/hr on the 2nd day and for 15 hours at rate of 32km/hr on the 3rd day. What is her average speed? 92.39 32 1 40 1 48 1 3 = ++ =HM Note: Whenever the frequency distribution are grouped (continuous), class marks of the class intervals are considered as and the above formula can be used asim ∑ = = n i i m f n HM 1 i where mi is the class mark of ith class.∑= = n i i fn 1 Relations among different means 1. If all the observations are positive we have the relationship among the three means given as: HMGMx ≥≥ 23   
  • 24. 2. For two observations GMHMx =* 3. HMGMx == if all observation are positive and have equal magnitude. ii. Median The median of a set of items (numbers) arranged in order of magnitude (i.e. in an array form) is the middle value or the arithmetic mean of the two middle values. We shall denote the median of by x~ .nxxx .,..,, 21 Median for Ungrouped Frequency Distribution We arrange the sample in ascending order of the variable of interest. Then the median is the middle value (if the sample size n is odd) or the average of the two middle values (if the sample size n is even). For ungrouped data the median is obtained by ⎪ ⎪ ⎪ ⎪ ⎩ ⎪ ⎪ ⎪ ⎪ ⎨ ⎧ + = ⎟ ⎠ ⎞ ⎜ ⎝ ⎛ +⎟ ⎠ ⎞ ⎜ ⎝ ⎛ ⎟ ⎠ ⎞ ⎜ ⎝ ⎛ + evenisnitemsofnumbertheif itemitem oddisnitemsofnumbertheifitem x nn n thth th ,,, 2 ,,, ~ 1 22 2 1 Example: Compute the medians for the following two sets of data A: 38 41 34 46 36 30 37 36 32 40. B: 28 32 36 30 31 37 36 80 32 34 31 Solution For A, the sample times in ascending order are: 30 32 34 36 36 37 38 40 41 46. The two middle values have been underlined, So the median time ( x~ ) = 5.36 2 3736 = + minutes. For B, the sample times in ascending order are: 28 30 31 31 32 32 34 36 36 37 80. The middle value has been underlined. So the median time ( x~ ) = 32 minutes. The median is easy to calculate for small samples and is not affected by an "outlier" - atypical value in the sample, e.g. the value 80 in the sample for route B. It is valid for ranked (ordinal) as well as interval/ratio (numeric) data. Median for Grouped Frequency Distribution For grouped data the median, obtained by interpolation method, is given by   24   
  • 25. Where lower class boundary of the median class=medL =F Sum of frequencies of all class lower than the median class (in other words it is the cumulative frequency immediately preceding the median class) Frequency of the median class and=medf Class width=W The median class is the class with the smallest cumulative frequency greater than or equal to 2 n . Example: Calculate the median for the following frequency distribution. Class Class Boundary Frequency Less than Cum. Freq (CF) 0 – 5 5 5 5 – 10 8 13 10 – 15 10 23 15 – 20 8 31 20 – 25 5 36 Solution: To obtain the median class we divide 36 by 2. That is, 18 2 36 2 ==n Thus the smallest cumulative frequency that contains 18 is 23. Hence the median class is the 3rd class which is 10 – 15. Therefore median is   Where 9.5, 13 sum of frequencies of all class lower than the 3rd class=medL =F 10, frequency of the 3rd class and,=medf =W 6, class width 5.12 35.9 10 1318 65.9~ = += ⎟ ⎠ ⎞ ⎜ ⎝ ⎛ − +=x iii. Mode The mode or the modal value is the most frequently occurring score/observation in a series and denoted by . A data set may not have a mode or may have more than one mode. A distribution is called a bimodal distribution if it has two data values that appear with the greatest frequency. If a distribution has more than two modes, then the distribution is multimodal. If a distribution has no modes, then the distribution is non-modal. xˆ Example: Consider the following data: Data X: 3, 4, 6, 12, 31, 8, 9, 8 the Mode ( ) = 8xˆ Data Y: 6, 8, 12, 13, 11, 12, 6 the Mode ( ) = 6 and 12xˆ Data Z: 2, 6, 3, 5, 7, 8, 12, 11 No Mode Note that in some samples there may be more than one mode or there may not be a mode. The mode is not a suitable measure of central tendency in these cases. 25   
  • 26. Mode for Grouped Frequency Distribution For grouped data, the mode is found by the following formula: WLx ⎟⎟ ⎠ ⎜⎜ ⎝ Δ+Δ += 21 1 mod ˆ ⎞⎛ Δ Where lower class boundary of the modal class=modL 11 ff −=Δ , 22 ff −=Δ =1f Frequency of the modal class =1f Frequency of the class immediately preceding the modal class =2f Frequency of the class immediately follows the modal class =W the class width The modal class is the class with the highest frequency in the distribution. Example: Find the mode for the frequency distribution of the birth weight (in kilogram) of 30 children given below. Weight 1.9-2.3 2.3-2.7 2.7-3.1 3.1-3.5 3.5-3.9 3.9-4.3 No. of children 5 5 9 4 4 3 Solution: 2.7- 3.1 is the modal class since it has the highest frequency and4591 =−=Δ 5492 =−=Δ 7.2mod =L 878.24.0* 54 4 7.2ˆ =⎟ ⎠ ⎞ ⎜ ⎝ ⎛ + +=x   The Relationship of the Mean, Median and Mode • In the case of symmetrical distribution; mean, median and mode coincide. That is mean=median = mode. However, for a moderately asymmetrical (non symmetrical) distribution, mean and mode lie on the two ends and median lies between them and they have the following important empirical relationship: Mean – Mode = 3(Mean - Median) ⇒ ( )xxxx ~3ˆ −=− Example In a moderately asymmetrical distribution, the mean and the median are 20 and 25 respectively. What is the mode of the distribution? Solution: Mode = 3median – 2mean = 3(25) – 2(20) = 75 – 40 = 35, hence the mode of the distribution will be 35. 4. MEASURES OF VARIATION (DISPERSION) Introduction and objectives of measuring variation We have seen that averages are representatives of a frequency distribution. But they fail to give a complete picture of the distribution. They do not tell anything about the spread or dispersion of observations within the distribution. Suppose that we have the distribution of yield (kg per plot) of two rice varieties from 5 plots each. 26   
  • 27. Variety 1: 45 42 42 41 40 Variety 2: 54 48 42 33 30 The mean yield of both varieties is 42 kg. The mean yield of variety 1 is close to the values in this variety. On the other hand, the mean yield of variety 2 is not close to the values in variety 2. The mean doesn’t tell us how the observations are close to each other. This example suggests that a measure of central tendency alone is not sufficient to describe a frequency distribution. Therefore, we should have a measure of spreads of observations. There are different measures of dispersion. Objectives of measuring variation • To describe dispersion (variability) in a data. • To compare the spread in two or more distributions. • To determine the reliability of an average. Note: The desirable properties of good measures of variation are almost identical with that of a good measure of central tendency. Absolute and relative measures Absolute measures of dispersion are expressed in the same unit of measurement in which the original data are given. These values may be used to compare the variation in two distributions provided that the variables are in the same units and of the same average size. In case the two sets of data are expressed in different units, however, such as quintals of sugar versus tones of sugarcane or if the average sizes are very different such as manager’s salary versus worker’s salary, the absolute measures of dispersion are not comparable. In such cases measures of relative dispersion should be used. A measure of relative dispersion is the ratio of a measure of absolute dispersion to an appropriate measure of central tendency. It is a unitless measure. Types of Measures of Variation i. The range and relative range Range(R) is defined as the difference between the maximum and minimum observations in a set of data. Range Maximum value Minimum value= − It is the crudest absolute measures of variation. It is widely used in the construction of quality control charts and description of daily temperature. Properties of range It is affected by extreme values. It does not take into account all observations. It is easy to calculate and simple to understand. It does not tell anything about the distribution of values in the set of data relative to some measures of central tendency. Relative range (RR) is defined as . . Range RR Max value Min value = + ii. The mean deviation and coefficient of mean deviation Mean deviation (MD) is the average of the absolute deviations taken from a central value, generally the mean or median. 1 1 | | | | n k i i i i i X X X X X f MD n n = = − − = = ∑ ∑ , 1 1 | | | | n k i i i i X iX X X X MD n n = = − − = = ∑ ∑ % % % f 27   
  • 28. Example: Calculate the mean deviation about the median and about the mean of the following scores of students in a certain test. 6,7,7,10,10 1 1.6 5 5 i X MD n = = = = = | | | 6 8| | 7 8|*2 |10 8|*2 8 k i iX X f− − + − + −∑ 1 | | | 6 7 | | 7 7 |*2 |10 7 |*2 7 1.4 5 5 k i i i X X X f MD n = − − + − + − = = = ∑ % % = Note: In case of grouped data, the mid-point of each class interval is treated as and we can use the above formula. Besides, X X MD MD≥ % Properties of mean deviation It is relatively simple to understand as compared to standard deviation. Its computation is simple. It is less affected by extreme values than standard deviation. It is better than the range and QD since it is based on all observations. It is not suitable for further statistical treatment. iii. Variance, standard deviation and coefficient of variation The variance is the average of the squares of the distance each value is from the mean. The symbol for the population variance is σ2 . Let 1 2, ,..., Nx x x be the measurements on N population units then, the population variance is given by the formula: 2 12 2 2 1 1 ( ) N iN N i i i i i x x x N N N μ σ = = = ⎛ ⎞ ⎜ ⎟ ⎝ ⎠− − = = ∑ ∑ ∑ , Where µ is population mean and N is population size Let 1 2, ,..., nx x x be the measurements on n sample units then, the sample variance is denoted by S2 , and its formula is 2 12 2 2 1 1 ( ) 1 1 n in n i i i i i x x x x n S n n = = = ⎛ ⎞ ⎜ ⎟ ⎝ ⎠− − = = − − ∑ ∑ ∑ Where x is the sample mean and n is the sample size. Standard deviation, denoted by σ or S, is the square root of the variance. That is, Population standard deviation 2 σ σ= and sample standard deviation 2 S S= . Example: For a newly created position, a manager interviewed the following numbers of applicants each day over a five-day period: 16, 19, 15, 15, and 14. Find the variance and standard deviation. Solution: 5 1 16 19 15 15 14 79 15.8 5 5 5 i i x x = + + + + = = = = ∑ 28   
  • 29. 5 2 2 2 2 1 ( ) (16 15.8) ... (14 15.8) 14.8 3.7 5 1 4 4 i i x x S = − − + + − ⇒ = = = = − ∑ Or ( ) 2 25 12 2 2 2 1 16 ... 14 6241 16 ... 14 1263 14.85 5 5 3.7 5 1 4 4 4 n i i i i x x S = = ⎛ ⎞ ⎜ ⎟ + +⎝ ⎠− + + − − = = = = − ∑ ∑ = For grouped frequency distribution, the formula for variance is 2 12 2 2 1 1 ( ) 1 1 k i ik k i i i i i i i f x f x x f x n S n n = = = ⎛ ⎞ ⎜ ⎟ ⎝ ⎠− − = = − − ∑ ∑ ∑ Where is the number of classes,k ix is the class mark of class i and 1 k i i n f = = ∑ Properties of variance The unit of measurement of the variance is the square of the unit of measurement of the observed values. It is one of its limitations. The variance gives more weight to extreme values as compared to those which are near to the mean value, because the difference is squared in variance. It is based on all observations in the data set. Properties of standard deviation Standard deviation is considered to be the best measure of dispersion and is used widely. There is, however, one difficulty with it. If the unit of measurement of variables of two series is not the same, then their variability cannot be compared by comparing the values of standard deviation. Uses of the variance and standard deviation The variance and standard deviations can be used to determine the spread of data, consistency of a variable and the proportion of data values that fall within a specified interval in a distribution. If the variance or standard deviation is large, the data is more dispersed. This information is useful in comparing two or more data sets to determine which is more (most) variable. Finally, the variance and standard deviation are used quite often in inferential statistics. Coefficient of variation (CV) The standard deviation is an absolute measure of dispersion. The corresponding relative measure is known as the coefficient of variation (CV). Coefficient of variation is used in such problems where we want to compare the variability of two or more different series. Coefficient of variation is the ratio of the standard deviation to the arithmetic mean, usually expressed in percent: *100% s CV x = A distribution having less coefficient of variation is said to be less variable or more consistent or more uniform or more homogeneous. 29   
  • 30. Example: Last semester, the students of two departments, A and B took Stat 276 course. At the end of the semester, the following information was recorded. Dept A Dept B Mean score 79 64 SD 23 11 Compare the relative dispersion of the two departments. 23 *100% *100 29.11% 79 A A A s CV x = = = , 11 *100% *100 17.19% 64 B B B s CV x = = = Since ACV > , the variation is department A is greater. Or, in department B the distribution of the marks is more uniform (consistent). BCV Exercise: The mean weight of 20 children was found to be 30 kg with variance of 16kg2 and their mean height was 150 cm with variance of 25cm2 . Compare the variability of weight and height of these children. iv. The standard scores A standard score is a measure that describes the relative position of a single score in the entire distribution of scores in terms of the mean and standard deviation. It also gives us the number of standard deviations a particular observation lie above or below the mean. Standard score, , x x X Z or Z S μ σ − − = = Where x is the value of the observation, / Xμ and / Sσ are the mean and standard deviation of the respectively. Interpretation: , , , Negative the observation lies below the mean If Z is Positive theobservation lies above the mean Zero the observationequals to the mean ⎧ ⎪ ⎨ ⎪ ⎩ Example: Two sections were given an exam in a course. The average score was 72 with standard deviation of 6 for section 1 and 85 with standard deviation of 5 for section 2. Student A from section 1 scored 84 and student B from section 2 scored 90. Who performed better relative to his/her group? Solution: Section 1: x = 72, S = 6 and score of student A from Section 1; A x = 84 Section 2: x = 85, S = 5 and score of student B from Section 2; B x = 90 Z-score of student A: 84 72 2 6 A A A A x X Z S − − = = = Z-score of student B: 90 85 1 5 B B B B x X Z S − − = = = From these two standard scores, we can conclude that student A has performed better relative to his/her section students because his/her score is two standard deviations above the mean score of selection 1 while the score of student B is only one standard deviation above the mean score of section 2 students. Exercise: A student scored 65 on a calculus test that had a mean of 50 and a standard deviation of 10; she scored 30 on a algebra test with a mean of 25 and a standard deviation of 5. Compare her relative positions on each test. 30   
  • 31. v. Skewness and kurtosis Skweness refers to lack of symmetry in a distribution. If a distribution is not symmetrical we call it skewed distribution. Note that for a symmetrical and unimodal distribution: Mean = median = mode Measure of skewness: 3 ˆx x s α − Pearsonian coefficient of skewness (Pcsk) defined as: = Interpretation: 3 0, 0, 0, the distribution isnegatively skewed If the distribution is positively skewed the distribution is symetrical α <⎧ ⎪ >⎨ ⎪=⎩ In moderately skewed distributions: Mode = mean- 3(mean-median) 3 3( )x x s α − ⇒ = % Note: in a negatively skewed distribution larger values are more frequent than smaller values. In a positively skewed distribution smaller values are more frequent than larger values. Exercise: If the mean, mode and s.d of a frequency distribution are 70.2, 73.6, and 6.4, respectively. What can one state about its skeweness? Kurtosis refers to the degree of peakedness of a distribution. When the values of a distribution are closely bunched around the mode in such a way that the peak of the distribution becomes relatively high, the distribution is said to be leptokurtic. If it is flat topped we call it platykurtic. A distribution which is neither highly peaked nor flat topped is known as a mesokurtic distribution (normal). 31